You are on page 1of 98

1. Which of the following processes is most likely to be associated with apoptosis?

A. Nucleosomal DNA laddering


B. Loss of plasma membrane integrity
C. Swelling of the nucleus
D. Endoreduplication

Correct answer is A. RATIONALE: Apoptosis is a type of programmed cell death involving


orderly, well-defined, controlled processes. One of the most prominent of these processes
involves the cleavage of chromatin at specific intervals defined by nucleosomal units. While it is
possible to have apoptosis without DNA laddering, the remaining choices do not imply
"controlled" processes at all.

2. Which of the following terms is defined as the net energy transferred to charged particles
per unit mass, at a point of interest, excluding radioactive energy loss and excluding
energy passed from one charged particle to another for photons?

A. Exposure
B. Collision kerma
C. Mass energy absorption coefficient
D. Absorbed dose

Correct answer is B. RATIONALE: This item requires knowledge of the definitions and
distinctions between four common physics terms.

3. The purpose of a phase II clinical trial is to:

A. estimate the toxicity of treatment to be tested.


B. estimate the efficacy of treatment to be tested.
C. estimate the preclinical toxicity of an experimental agent.
D. determine a safe starting dose of an experimental agent.

Correct answer is B. RATIONALE: A Phase II trial is performed to determine if a drug or


treatment has activity against a specific type of tumor.

4. Which of the following chemotherapy agents is most effective against soft tissue
sarcoma?

A. Thiotepa
B. Cyclophosphamide
C. Dactinomycin
D. Doxorubicin

Correct answer is D. RATIONALE: Doxorubicin has been a mainstay of chemotherapy for


advanced soft tissue sarcomas, with a 20-30% response rate. More recent studies also have
demonstrated the efficacy of ifosfamide.

Radiation Oncology In-Training Exam 2007 3


5. What number of axillary lymph nodes removed for sampling during a level I / II node
dissection for breast cancer most likely would require postoperative radiation therapy to
the axilla?

A. 02
B. 06
C. 08
D. 10

Correct answer A. RATIONALE: The number of chest wall recurrences increased in cases
where the number of axillary lymph nodes removed for sampling was less than 6. The minimum
number of axillary lymph nodes to be removed in a level I/II node dissection is 6. Reference:
Recht A, et al. Regional nodal failure after conservative surgery and radiotherapy for early-stage
breast carcinoma. Journal of Clinical Oncology. 1991;9(6):988-996.

6. Which of the following types of cancer was associated with a statistically significant
improvement in survival for patients who had one to three solitary brain metastases and
received whole brain radiation therapy with the addition of stereotactic radiosurgery,
according to results of the RTOG 9508 phase III trial?

A. Breast cancer
B. Renal cell cancer
C. Non-small cell lung cancer
D. Melanoma

Correct answer is C. RATIONALE: Based on the RTOG 9508 phase III trial results, patients
with non-small cell lung cancer (NSCLC)/small cell cancer (SCC) had improved median survival
with the addition of stereotactic radiosurgery (SRS) to whole brain radiation therapy (WBRT) on
subset analysis. This is the basis for the ongoing RTOG 0320 phase III trial.

7. In RTOG 8501, what was the 5-year overall survival rate for patients who had
esophageal cancer and received radiation therapy alone?

A. 00%
B. 10%
C. 25%
D. 35%

Correct answer is A. RATIONALE: This is a landmark study that established the superiority of
combined chemoradiation therapy to radiation therapy alone for esophageal cancer. Reference:
Herskovic. New England Journal of Medicine (NEJM). 1992;326:1593.

4 American College of Radiology


8. Which of the following tumor stages is correctly matched to the appropriate prostate
cancer involvement or invasion?

A. Stage T2b is associated with tumor involvement of more than one half of one lobe
but not both lobes of the prostate.
B. Stage T2c is associated with nonpalpable tumor involvement of both lobes of the
prostate via needle biopsy.
C. Stage T3a is associated with tumor invasion of the prostate apex.
D. Stage T4 is associated with tumor invasion of the seminal vesicles.

Correct answer is A. RATIONALE: The correct answer for option B is T1c (not T2c), option C
is T2 (not T3a), and option D is T3b (not T4). Reference: AJCC Cancer Staging Manual (6th
edition).

9. According to GOG 122 (Randall), the survival rate of patients who have surgical
stage III-IV endometrial cancer and receive adjuvant abdominopelvic irradiation
compared to adjuvant chemotherapy is:

A. worse.
B. similar.
C. improved.
D. not able to be analyzed.

Correct answer is A. RATIONALE: In the GOG 122 trial, patients treated with adjuvant
abdominopelvic irradiation had a worse survival (42% vs. 55%, p < 0.01) compared to patients
treated with adjuvant chemotherapy. Reference: Randall, et al. Journal of Clinical Oncology.
2006;24:36-44.

10. A well-lateralized, stage T3N0 squamous cell carcinoma of the nasopharynx has at least
a 15% risk of metastasis to the:

A. ipsilateral parotid lymph nodes.


B. ipsilateral submandibular lymph nodes.
C. posterior triangle lymph nodes bilaterally.
D. occipital lymph nodes bilaterally.

Correct answer is C. RATIONALE: An important concept in the management of


nasopharyngeal cancer is the need for elective irradiation of the level V nodes bilaterally,
regardless of the status of the neck. The lymph nodes in the other areas are not routinely
treated in a patient with a stage N0 neck.

11. What is the second leading cause of lung cancer?

A. Exposure to arsenic
B. Exposure to asbestos
C. Exposure to benzene
D. Exposure to radon

Correct answer is D. RATIONALE: Radon is the second leading single agent responsible for
causing lung cancer; asbestos is responsible for only 3% to 4% of cases.

Radiation Oncology In-Training Exam 2007 5


12. Which of the following chromosomal translocations is most commonly associated with
Burkitt lymphoma?

A. t(8;14)
B. t(9;22)
C. t(11;14)
D. t(14;18)

Correct answer is A. RATIONALE: Option B is the correct answer for chronic myelogenous
leukemia (CML), option C is for mantle cell lymphoma, and option D is for follicular lymphoma.

13. What is the most common secondary malignant neoplasm inside the radiation field for
patients with retinoblastoma?

A. Squamous cell carcinoma


B. Non-Hodgkin lymphoma
C. Fibrosarcoma
D. Osteosarcoma

Correct answer is D. RATIONALE: It is important to know that common secondary malignant


neoplasms inside the radiation field for patients with retinoblastoma are osteosarcoma,
fibrosarcoma, or spindle cell sarcoma, with osteosarcoma being the most common.

14. Apoptosis is most likely to be involved in which of the following normal


biological/developmental processes?

A. Fertility
B. Hair growth
C. Muscle hypertrophy
D. Tooth formation

THIS ITEM WAS NOT SCORED. RATIONALE: Hair growth is due to continuous cycles of cell
division and programmed cell death, i.e., apoptosis, in specialized epithelial cells at the root of
the hair follicle. The collection of epithelial cells is often interspersed with melanocytes, which
give rise to hair color. The balance of cell division and controlled apoptosis results in
lengthening of hair, which may be disrupted by radiation or chemotherapy, resulting in alopecia.
Apoptosis of eggs would result in infertility, not fertility.

15. Which of the following terms is defined as the energy absorbed per unit mass for
photons?

A. Exposure
B. Collision kerma
C. Mass energy absorption coefficient
D. Absorbed dose

Correct answer is D. RATIONALE: This item requires knowledge of the definitions and
distinctions between four common physics terms.

6 American College of Radiology


16. Three different lung cancer radiation treatments are being assessed for
maximum-tolerated dose. This study is an example of what type of clinical trial?

A. Phase I
B. Phase II
C. Phase III
D. Pilot

Correct answer is A. RATIONALE: The objective of a phase I trial is to determine an


appropriate dose for use in a phase II or phase III trial.

17. Which of the following factors is the most important predictor of disease-free survival
and overall survival in patients with breast cancer?

A. Tumor size
B. Mitotic index
C. Axillary lymph node status
D. Histopathologic tumor grade

Correct answer is C. RATIONALE: All of these factors are shown to be prognostic for disease-
free survival (DFS) and overall survival (OS) by multivariate analysis. However, axillary lymph
node status is the most powerful predictor of these factors. Reference: Fitzgibbons, PL, et al.
Prognostic factors in breast cancer. College of American Pathologists Consensus Statement.
1999. Archives of Pathology and Laboratory Medicine. 2000;124:966-978.

18. Which of the following neurological cognitive tests is most appropriate for measuring a
patient's memory?

A. Hopkins Verbal Learning Test (HVLT)


B. Controlled Oral Word Association (COWA) Test
C. Trail Making Test, Part A
D. Grooved Pegboard Test

Correct answer is A. RATIONALE: Memory is measured by the Hopkins Verbal Learning Test
(HVLT).

19. Which of the following diagnostic tests is most useful for determining the tumor stage of
esophageal cancer?

A. Esophagogastroduodenoscopy
B. Endoscopic ultrasound
C. PET scan
D. CT scan

Correct answer is B. RATIONALE: Endoscopic ultrasound (EUS) is the most useful diagnostic
test for determining the tumor (T) stage of esophageal cancer.

Radiation Oncology In-Training Exam 2007 7


20. What is the best estimate of the α/β ratio for prostate cancer?

A. <3 Gy
B. 3 Gy to 5 Gy
C. 6 Gy to 8 Gy
D. >8 Gy

Correct answer is A. RATIONALE: Both preclinical and clinical evidence suggests that
prostate cancer has a low alpha/beta (α/β) ratio, more like late-responding normal tissues.
References: Brenner, DJ, et al. International Journal of Radiation Oncology, Biology, Physics.
2002 (Jan 1);52(1):6-13.

21. Which of the following studies compared surgery alone versus surgery plus
external-beam pelvic irradiation in patients with early-stage endometrial cancer and
required all patients to undergo surgical staging with pelvic lymphadenectomy?

A. GOG 99 (Keys)
B. GOG 33 (Morrow)
C. PORTEC (Creutzberg)
D. Norwegian (Aalders)

Correct answer is A. RATIONALE: Of the 3 published trials, the sole trial to require pelvic
lymphadenectomy was the GOG 99 trial. In the Norwegian and PORTEC trials, all patients
underwent hysterectomy alone without surgical evaluation of the pelvic and/or paraaortic lymph
nodes. GOG 33 investigated the relationship between surgical-pathological risk factors and
outcome in clinical stage I and II carcinoma of the endometrium. Reference: Keys, et al.
Gynecologic Oncology. 2004;92:744-51.

22. Which of the following findings is most likely to be caused by loss of function of right
cranial nerve XII?

A. Deviation of the tongue to the left side with protrusion


B. Deviation of the tongue to the right side with protrusion
C. Loss of sweet taste on the right side of the tongue
D. Loss of salty taste on the right side of the tongue

Correct answer is B. RATIONALE: Deviation of the tongue to the ipsilateral side is the major
finding in a patient with loss of function of cranial nerve XII.

23. Which of the following statements about video-assisted thoracic surgery for patients with
lung cancer is FALSE?

A. The postoperative period is shorter.


B. It decreases the risk of intraoperative bleeding.
C. It is a viable alternative to thoracotomy in patients who are frail.
D. Locoregional recurrence is increased.

Correct answer is D. RATIONALE: The regional recurrence and long-term survival with
video-assisted thoracic surgery (VATS) and lymph node dissection are equal to open lung
resection.

8 American College of Radiology


24. According to GOG 99 (Keys), which of the following sites is most likely to be associated
with cancer recurrence in a patient who has early-stage endometrial cancer and
undergoes TAH-BSO without adjuvant irradiation?

A. Liver
B. Vaginal vault
C. Pelvic lymph nodes
D. Paraaortic lymph nodes

Correct answer is B. RATIONALE: In the GOG 99 trial comparing surgery alone with surgery
plus adjuvant pelvic irradiation in patients with stage I-II endometrial cancer, the most common
site of failure in the surgery alone arm was in the pelvis. However, of these failures, nearly
three fourths were in the vaginal vault. Reference: Keys, et al. Gynecologic Oncology.
2004;92:744-51.

25. According to the IRS-I and IRS-II data analyses, which of the following primary sites of
rhabdomyosarcoma is associated with the highest risk of lymph node metastasis?

A. Orbit
B. Prostate
C. Lower extremity
D. Parameningeal

THIS ITEM WAS NOT SCORED. RATIONALE: Analysis of IRS-I and IRS-II data revealed that
among all of the sites listed, the prostate had a 42% risk of lymph node metastasis compared to
other sites, such as the orbit (0%), the non-orbit head and neck sites (8%), and the lower
extremity (9%). Lymph node metastasis has prognostic significance.

26. Which of the following laboratory tests is most appropriate for determining if a patient
with acromegaly has responded favorably to radiation therapy?

A. Serum glucose level


B. Growth hormone (GH)
C. Thyroid stimulating hormone (TSH)
D. Insulin-like growth factor-1 (IGF-1)

Correct answer is D. RATIONALE: Insulin-like growth factor-1 (IGF-1) is a better test than
growth hormone (GH).

27. What is the daughter product associated with electron capture from 192
77 Ir ?

A. 192
76 Os
192
B. 78 Os
C. 192
76 Pt
192
D. 78 Pt

THIS ITEM WAS NOT SCORED. RATIONALE: This item requires knowledge of balancing
191 1 192 192 0
n Pt
nuclear reaction equations for this reaction. 77 Ir + 0 → ( 77 Ir ) → 78 + −1 β +γ (Beta
191 1 192 0 192
n
decay) 77 Ir + 0 → ( 77 Ir + −1 β) → 76
Os + γ (Electron capture)

Radiation Oncology In-Training Exam 2007 9


28. Which of the following soft tissue sarcomas is most likely to be found in a patient with
neurofibromatosis, type 1?

A. Liposarcoma
B. Fibrosarcoma
C. Leiomyosarcoma
D. Malignant peripheral nerve sheath tumor

Correct answer is D. RATIONALE: Approximately 5% of patients with neurofibromatosis will


develop malignant peripheral nerve sheath tumor. Reference: DeVita, et al. Sarcomas of the
soft tissue and bone. Cancer Principles and Practice of Oncology. 5th edition.

29. According to an NSABP cumulative analysis, which of the following scenarios about the
risk of locoregional breast cancer recurrence after treatment with mastectomy,
chemotherapy and no chest wall irradiation is true?

A. A higher risk of local failure is associated with patients who have one to three
positive lymph nodes and an increased tumor size.
B. Patients with the same tumor size but an increase in the number of positive lymph
nodes will not have an increased risk of local failure.
C. The rate of isolated regional failure is 3% in patients with one to three positive lymph
nodes and a tumor size of <2 cm.
D. The number of lymph nodes examined was not predictive of regional cancer
recurrence.

THIS ITEM WAS NOT SCORED. RATIONALE: The number of regional failures increases as
does the tumor size and the number of positive axillary nodes found. Reference: Taghain, AG,
et al. Pattern of locoregional and distant failure in patients with breast cancer treated with
mastectomy and chemotherapy (+/- tamoxifen) without radiation: results from five NSABP
randomized trials. International Journal of Radiation Oncology, Biology, Physics.
2001;51(1):106. Perez, CA, Brady, LW, Halperin, EC. Schmidt-Ullrich, RK, eds. Principles and
Practice of Radiation Oncology, 4th edition. Philadelphia: Lippincott Williams & Wilkins
Publishers, 2004; p 1411.

30. Which of the following survival times is most likely to be associated with a patient who
has a class II brain metastasis, based on the Radiation Therapy Oncology Group
(RTOG) recursive partitioning analysis (RPA)?

A. 10.2 months
B. 07.1 months
C. 04.2 months
D. 02.3 months

Correct answer is C. RATIONALE: Based on the Radiation Therapy Oncology Group (RTOG)
recursive partitioning analysis (RPA) for brain metastases, 4.2 months is the correct answer.

10 American College of Radiology


31. Radiation-induced cell death in carcinomas typically occurs immediately after attempting
to traverse which phase of the cell cycle?

A. G0
B. G1
C. G2
D. M

Correct answer is D. RATIONALE: The primary mode of cell death in most epithelial
malignancies and the cell lines derived from such tumors after exposure to radiation doses in
the therapeutic range is either "mitotic catastrophe" or "postmitotic death." The treated cancer
cells go into mitosis with damaged DNA/chromosomes, which then leads to nondisjunction,
unequal distribution of chromosomes to daughter cells, and numerous translocations or
chromosomal loss. The doomed cell either dies without successfully completing mitosis or soon
after the aberrant mitosis. The implication is that non-cycling cells tend to be more
radioresistant.

32. Which of the following radionuclides has a decay constant (λ) of 0.011628 per day?
103
A. Pd
125
B. I
131
C. Cs
198
D. Au

Correct answer is B. RATIONALE: 125I has a decay constant (λ) of 0.011628 per day.

33. What is the recommended dose for definitive radiation treatment of esophageal cancer,
according to the Intergroup 0123 study?

A. 45 Gy
B. 50.4 Gy
C. 54 Gy
D. 59.4 Gy

Correct answer is B. RATIONALE: In the Intergroup 0123 study, patients were randomized to
64 vs. 50.4 Gy. The lower dose arm had a higher median survival of 17.5 months vs. 12.9
months. Reference: Journal of Clinical Oncology. 2002;20:1167–74.

34. Which of the following statements about hypofractionated radiation therapy for prostate
cancer is FALSE?

A. According to a randomized trial comparing standard fractionation versus


hypofractionation, hypofractionation results in more severe acute GI and GU toxicity.
B. The slow growth rate of prostate cancer is associated with high sensitivity to
increased fraction size.
C. A radiobiological model suggests that HDR brachytherapy may achieve higher
biologically effective doses.
D. At 5 years, hypofractionation using 70 Gy at 2.5 Gy per fraction produced excellent
local control.

Correct answer is A. RATIONALE: Hypofractionated radiation therapy at 2.7 Gy per fraction


was well tolerated acutely. Reference: Pollack, et al. International Journal of Radiation
Oncology, Biology, Physics. 2006 (Feb 1);64(2):518-26.

Radiation Oncology In-Training Exam 2007 11


35. Which of the following antigens is a T-cell marker?

A. CD7
B. CD20
C. CD30
D. CD40

THIS ITEM WAS NOT SCORED. RATIONALE: T-cell lineage is evident from the presence of
T-cell markers (CD3, CD2, and CD7).

36. Which of the following depths of invasion is considered to be the threshold for elective
irradiation of the regional lymph nodes in a patient with squamous cell carcinoma of the
oral tongue?

A. 01.0 mm
B. 03.0 mm
C. 10.0 mm
D. 15.0 mm

Correct answer is B. RATIONALE: A large number of studies recommend a threshold value of


2 mm to 5 mm depth of invasion for elective treatment of the regional lymph nodes in this
setting.

37. Which of the following statements about adenocarcinoma of the lung is true?

A. It is the most common histologic subtype.


B. It is treated differently than squamous cell carcinoma.
C. It is less likely to metastasize to the brain than other histologies.
D. It is rarely associated with TTF-1.

Correct answer is A. RATIONALE: Adenocarcinoma has surpassed squamous cell carcinoma


of the lung as the most common histological subtype.

38. According to the National Comprehensive Cancer Network (NCCN) guidelines, which of
the following radiation dose ranges would be most appropriate for treatment of
intermediate-grade non-Hodgkin lymphoma?

A. 10 Gy to 15 Gy
B. 30 Gy to 40 Gy
C. 50 Gy to 55 Gy
D. 60 Gy to 70 Gy

Correct answer is B. RATIONALE: National Comprehensive Cancer Network (NCCN)


guidelines specify 30 Gy to 36 Gy or 40 Gy in the case of bulky disease.

12 American College of Radiology


39. A 9-month-old infant has a mass in the left flank, periorbital ecchymosis, and bone
metastasis to the right femur. Biopsy results are consistent with neuroblastoma. What
cancer stage would be most appropriate for this patient's condition?

A. Stage 2
B. Stage 3
C. Stage 4
D. Stage 4S

Correct answer is C. RATIONALE: If there were no presence of bone metastases, this infant’s
disease would be classified as cancer stage 4S. However, bone involvement (not bone
marrow) upstages the infant to cancer stage 4, and treatment is entirely different.

40. Hypoxia adversely affects the treatment and management of cancers because it:

A. increases blood flow to the tumor.


B. impedes adequate blood flow to the tumor.
C. activates molecular pathways contributing to resistance.
D. causes dyspnea and tachypnea in the patient.

Correct answer is C. RATIONALE: Hypoxia leads to activation of potent transcription factors,


such as HIF-1. This, in turn, activates numerous downstream targets, the majority of which
contribute to cancer cell survival. Hypoxia within the tumors is detectable even when blood
oxygen levels remain normal. Tumors often show aberrant perfusion patterns and do not
respond to physiological cues to the same degree as normal tissues.

41. For permanent implants with the same activity, which of the following radionuclides
yields the greatest absorbed dose to tissue?
103
A. Pd
125
B. I
131
C. Cs
198
D. Au

Correct answer is B. RATIONALE: Absorbed dose is directly proportional to the half-life of the
radionuclide.

42. What is the best method for minimizing bias in a national trial comparing two radiation
therapy regimens?

A. Randomization
B. Patient stratification
C. Univariate analysis of prognostic factors
D. Multivariate analysis of prognostic factors

Correct answer is A. RATIONALE: The aim of randomization is to minimize bias.

Radiation Oncology In-Training Exam 2007 13


43. Which of the following primary bone tumors is most common?

A. Ewing sarcoma
B. Malignant giant cell tumor
C. Chondrosarcoma
D. Osteosarcoma

Correct answer is D. RATIONALE: Osteosarcoma is the most common malignant primary


bone tumor (excluding multiple myeloma).

44. For the following trials, what is the correct isolated locoregional failure rate over a
10-year period for a premenopausal patient with one to three positive lymph nodes who
receives postmastectomy chemotherapy and tamoxifen with no radiation therapy?

A. NSABP (5-study analysis), 24%


B. Danish trial (82b), 30%
C. IBCSG, 35%
D. Vancouver, BC trial, 38%

Correct answer is B. RATIONALE: The 10-year published results are as follows: 33% for the
Vancouver, BC, trial; 13% for the NSABP trials; and 20% for the IBCSG trial.

45. Which of the following PIV/TV ratios (conformality indexes) would be most appropriate
when using conformal stereotactic radiosurgery to treat a patient with meningioma,
according to the RTOG 9005 trial treatment parameters?

A. 1.5
B. 2.5
C. 3.5
D. 4.5

Correct answer is A. RATIONALE: Extrapolating from the treatment parameters used in the
RTOG 9005 trial, the PIV/TV ratio should be <2.0 to minimize complications.

46. Which of the following types of esophageal cancer has had the greatest increase in
incidence over the past 40 years?

A. Squamous cell carcinoma in Caucasian men


B. Squamous cell carcinoma in African American men
C. Adenocarcinoma in Caucasian men
D. Adenocarcinoma in African American men

Correct answer is C. RATIONALE: Over the past 40 years, there has been a very dramatic
increase in esophageal adenocarcinoma in Caucasian men, which has not yet been able to be
explained. Reference: Devesa, et al. Cancer. 1998;83:2049-53.

14 American College of Radiology


47. Which of the following complications is NOT associated with a radical prostatectomy?

A. Proctitis
B. Erectile dysfunction
C. Urinary incontinence
D. Anastomotic stricture

Correct answer is A. RATIONALE: Proctitis is a known complication of radiation therapy.

48. Which of the following FIGO stages is associated with a patient who has endometrial
cancer with extension to the cervical stroma?

A. IC
B. IIA
C. IIB
D. IIIA

Correct answer is C. RATIONALE: According to the current FIGO staging system for uterine
tumors, extension to the cervical stroma is classified as stage IIB. Extension to the cervical
glands only is classified as stage IIA.

49. What is the risk of metastasis to the lymph nodes in a patient with stage T2N0
squamous cell carcinoma of the lateral aspect of the oral tongue?

A. There is a greater risk of metastasis to the retropharyngeal nodes than to the


mid-jugular nodes.
B. There is a greater risk of metastasis to the superior jugular nodes than to the
submandibular nodes.
C. There is a similar risk of metastasis to the retropharyngeal nodes as to the
mid-jugular nodes.
D. There is a similar risk of metastasis to the submandibular nodes as to the
mid-jugular nodes.

Correct answer is D. RATIONALE: An important concept in both surgical and radiotherapeutic


management of oral tongue cancer is parallel first echelon drainage to the submandibular
(level I) and mid-jugular lymph nodes.

50. Which of the following factors is NOT an independent prognostic indicator of non-small
cell carcinoma of the lung?

A. Age
B. Gender
C. Performance status
D. Weight loss

THIS ITEM WAS NOT SCORED.

Radiation Oncology In-Training Exam 2007 15


51. Which of the following factors is included in the international prognostic index (IPI) for
non-Hodgkin lymphoma?

A. Histology
B. Weight loss
C. Lactate dehydrogenase (LDH) level
D. Erythrocyte sedimentation rate (ESR)

Correct answer is C. RATIONALE: The international prognostic index (IPI) consists of stage,
age, performance status, LDH, and number of extranodal sites.

52. What percent of patients with neuroblastoma have metastases at diagnosis?

A. 10%
B. 25%
C. 60%
D. 80%

Correct answer is C. RATIONALE: Overall, 60% of patients with neuroblastoma have


metastases at diagnosis. The knowledge of this percentage is important because it justifies the
need for routine metastatic workup of patients with the disease.

53. Which of the following factors most adversely affects patients receiving radiation therapy
for squamous cell carcinoma of the head and neck?

A. Concurrent HPV infection


B. Concurrent tobacco use
C. Gender of the patient
D. Grade of the cancer

Correct answer is B. RATIONALE: Continued tobacco use by a patient undergoing radiation


therapy for head and neck malignancies is well-documented to be associated with poorer tumor
control rates, increased side effects, and increased need for treatment breaks. Unlike
herpesvirus or HIV infections, human papilloma virus (HPV) infections have not been shown to
be associated with increased treatment-related toxicities.

54. Which of the following forms of radiation demonstrates a Bragg peak in matter?

A. Protons
B. Gamma–rays
C. Neutrons
D. Electrons

Correct answer is A. RATIONALE: This item requires knowledge of the Bragg Peak.

16 American College of Radiology


55. Which of the following pathological findings is associated with patients who have
meningioma?

A. Verocay bodies
B. Psammoma bodies
C. Small, round blue cells
D. Flexner-Wintersteiner rosettes

Correct answer is B. RATIONALE: Psammoma bodies are associated with meningiomas.

56. According to Intergroup Trial 0116, adjuvant chemotherapy and radiation therapy after
gastrectomy were more likely than gastrectomy alone to:

A. result in an unacceptable level of toxicity in the majority of patients.


B. result in a statistically significant decrease in the rate of distant metastases.
C. result in a statistically significant improvement in overall patient survival.
D. be most beneficial after D2 lymph node dissections.

Correct answer is C. RATIONALE: Overall survival increased from 41% to 50%, with a
significant p value. Reference: New England Journal of Medicine (NEJM).
2002;345(10):725-30.

57. Which of the following statements about post-prostatectomy radiation therapy used in
the EORTC 22911 Trial is true?

A. It resulted in more frequent grade 2 or grade 3 late effects.


B. It improved local control, biochemical progression-free survival, and overall patient
survival.
C. It was beneficial for patients who had prostate cancer with positive lymph nodes.
D. Hormone therapy was added to the treatment regimen for patients with positive
lymph nodes.

Correct answer is A. RATIONALE: There is no overall survival benefit yet. Patients with
positive lymph nodes were not included, and hormonal therapy was not added.

58. Which of the following assays of normal tissue radiation response directly depends on
the clonogenicity of individual cells?

A. LD50
B. Pig skin reaction
C. Spleen colony
D. 51Cr clearance

Correct answer is C. RATIONALE: Only the spleen colony assay of Till and McCulloch
depends on the colony-forming ability of individual bone marrow stem cells. All of the other
assays depend on measuring a tissue structural or functional endpoint as a surrogate marker for
cell survival, and thus, are non-clonogenic.

Radiation Oncology In-Training Exam 2007 17


59. For 137Cs, a 1-inch-thick lead filter (HVL = 0.635 cm) will transmit about what fraction of
the incident radiation striking it?

A. 03%
B. 06%
C. 12%
D. 25%

Correct answer is B. RATIONALE: The 1-inch-thick shield constitutes four half-value layers.
Hence, (.5)*(.5)*(.5)*(.5) = 0.0625 or 6.25%.

60. Which of the following symptoms is most common in patients who present with
endometrial cancer?

A. Pelvic pain
B. Low back pain
C. Vaginal bleeding
D. Hematuria

Correct answer is C. RATIONALE: The most common presenting symptom in patients with
endometrial cancer is painless vaginal bleeding. Other symptoms, including pain and
hematuria, are signs of more advanced disease and are uncommon at initial presentation.

61. Which of the following stages of oropharyngeal squamous cell carcinoma is associated
with the lowest risk of contralateral cervical lymph node metastases?

A. T2N1 with the primary tumor confined to the tonsillar fossa


B. T1N1 involving the most lateral section of the soft palate
C. T1N1 confined to the glossal-tonsillar sulcus
D. T1N0 involving the lateral aspect of the base of tongue

Correct answer is A. RATIONALE: There are now multiple papers documenting a less than 5%
recurrence rate in the contralateral lymph nodes following radiation therapy to the ipsilateral
nodes (no treatment to the contralateral nodes) in patients with T1-T2, N0-N2b stage small cell
carcinoma that is confined to the tonsillar fossa. The other answers describe situations where
the risk of contralateral neck disease is clearly greater than 15%.

62. Which of the following statements about the use of amifostine concurrently with thoracic
external-beam radiation therapy is true?

A. Pneumonitis is decreased.
B. Esophagitis is increased.
C. Xerostomia and mucositis are increased.
D. Normal lung cells do not take up amifostine.

Correct answer is A. RATIONALE: Amifostine treatment before lung irradiation protects the
lung alveolocapillary integrity and has been found to have a protective effect in the acute phase
of radiation lung injury.

18 American College of Radiology


63. CHOP chemotherapy contains which of the following drugs?

A. Vinblastine
B. Hydroxyurea
C. Prednisone
D. Carmustine

Correct answer is C. RATIONALE: CHOP chemotherapy includes cyclophosphamide,


doxorubicin, Oncovin, and prednisone.

64. Which of the following types of abdominal tumor most commonly occurs in a 2-year-old
child?

A. Neuroblastoma
B. Rhabdomyosarcoma
C. Non-Hodgkin lymphoma
D. Wilms tumor

THIS ITEM WAS PARTIALLY SCORED (BOTH OPTIONS A & D SCORED CORRECT).
RATIONALE: Wilms tumor is the most common abdominal tumor that occurs during childhood,
followed closely by neuroblastoma. The median age at diagnosis of neuroblastoma is about
2 years of age, and the median age at diagnosis of Wilms tumor is 3.5 years of age.

65. In the absence of cell cycle effects and proliferation, as the dose per fraction of x-rays is
reduced from approximately 2 Gy to 0.2 Gy, the total dose that yields a 5% probability of
a normal tissue complication would be expected to:

A. remain unchanged.
B. continue to increase.
C. decrease to a point, and then remain unchanged.
D. increase to a point, and then remain unchanged.

Correct answer is D. RATIONALE: In the absence of cell cycle effects and/or cell proliferation,
the loss of additional sparing with increasing fractionation of the total dose is a reflection of the
fact that tissue dose response curves have non-zero initial slopes.

66. For absorbed dose from heterogeneity corrections calculated from CT images, a table is
required that converts Hounsfield units to which of the following physical properties of
the medium imaged?

A. Mass number
B. Atomic number
C. Electron density
D. Physical density

Correct answer is C. RATIONALE: Absorbed dose is directly proportional to electron density.

Radiation Oncology In-Training Exam 2007 19


67. Which of the following equations is used to determine a positive predictive value?

A. True positive / (True positive + false negative)


B. True positive / (True positive + false positive)
C. True negative / (True negative + false negative)
D. True negative / (True negative + false positive)

Correct answer is B. RATIONALE: By definition, the positive predictive value is true positive /
true + false positives.

68. Which of the following treatment outcomes is associated with patients who have DCIS,
according to prospective randomized trials?

A. In NSABP B-17, the actuarial rate of breast cancer recurrence after 12 years in
patients who received surgery only was 24%.
B. In NSABP B-17, the actuarial rate of breast cancer recurrence after 12 years in
patients who received surgery plus breast irradiation was 16%.
C. In NSABP B-24, breast irradiation reduced the risk of breast cancer recurrence in all
subgroups except for the patients with positive surgical margins.
D. In EORTC 10853, the subtype of DCIS histology had no effect on patients' risk for
breast cancer recurrence.

Correct answer is B. RATIONALE: In NSABP B-17, the actuarial rate of breast recurrence was
32% in the group not receiving radiation therapy. I n NSABP B-24, radiation reduced the risk of
breast recurrence in all subgroups, including the group with positive surgical margins. EORTC
reported the relative risk of recurrence was higher for cribriform and solid histologic subtypes
(RR 2.39) vs. clinging/micropapillary subtypes (RR 2.25). References: Burstein, HJ, et al.
Ductal carcinoma in situ of the breast. New England Journal of Medicine (NEJM).
2004;350:1430-144. Bjker N, et al. Breast-conserving treatment with or without radiotherapy in
ductal carcinoma-in-situ: ten year results of European Organization for Research and Treatment
of Cancer Randomized Phase III Trial 10853 – A study by the EORTC Breast Cancer
Cooperative Group and EORTC Radiotherapy Group. Journal of Clinical Oncology.
2006;24(21):3381-3387.

69. Which of the following radiation doses per fraction would be most appropriate for a
patient with malignant meningioma?

A. 45 Gy/25 fractions
B. 50.4 Gy/28 fractions
C. 54 Gy/30 fractions
D. 59.4 Gy/33 fractions

Correct answer is D. RATIONALE: Since this is a malignant tumor, a radiation dose of 59.4 Gy
would be appropriate.

20 American College of Radiology


70. Which of the following assays measures the reproductive capacity of individual cells?

A. TD50 limiting dilution


B. TCD50 tumor control
C. Clonogenic survival
D. MTT

Correct answer is C. RATIONALE: Only clonogenic survival assay measures the long-term
reproductive capability of cells (i.e., the ability to divide enough times to form colonies). The
other assays measure viability but not reproductive capability. It is argued that reducing the
reproductive capability of cancer cells is more clinically meaningful than achieving a specific
degree of cell killing.

71. Which of the following lymph nodes is NOT commonly associated with prostate cancer
metastases?

A. Inguinal
B. External iliac
C. Presacral
D. Obturator

Correct answer is A. RATIONALE: Inguinal lymph node metastasis from prostate cancer is not
common.

72. Which of the following FIGO stages is associated with a patient who has endometrial
cancer with extension through the entire myometrium and involvement of the surface of
the uterine serosa?

A. IC
B. IIB
C. IIIA
D. IIIC

Correct answer is C. RATIONALE: According to the current FIGO staging system for uterine
tumors, extension to the uterine serosa is classified as stage IIIA. Stage IIIA also includes
involvement of the adnexa and/or positive peritoneal cytology.

73. Image intensity in a uniform region of a nuclear medicine image follows Poisson
statistics. If the number of photons in a typical voxel increases by a factor of 4, then the
signal-to-noise ratio (value divided by the standard deviation) of the number of photons
in the group of similar voxels will:

A. increase by a factor of 4.
B. increase by a factor of 2.
C. decrease by a factor of 4.
D. decrease by a factor of 2.

Correct answer is B. RATIONALE: In Poisson statistics, the standard deviation is proportional


to the square root of the number of counted events. Thus, nx4 photons would have a standard
deviation of the sqrt (nx4) or sqrt(n)x2. The SNR = (nx4)/(sqrt(n)x2) = sqrt (n)x2 versus n/sqrt(n)
= sqrt(n) for the original photon counts.

Radiation Oncology In-Training Exam 2007 21


74. Which of the following treatment techniques is NOT an effective method for providing
expedient palliation for an endobronchial lesion?

A. Photodynamic therapy
B. 125I LDR therapy
C. 192Ir HDR therapy
D. Endobronchial stent placement

Correct answer is B. RATIONALE: 125I LDR therapy is not an effective method for providing
expedient palliation for an endobronchial lesion.

75. Which of the following histologies is most likely to be associated with primary CNS
lymphoma?

A. Burkitt
B. Follicular
C. Hodgkin disease
D. Diffuse large B-cell

Correct answer is D. RATIONALE: Most primary CNS lymphomas are diffuse large B-cell,
immunoblastic or lymphoblastic lymphomas.

76. Which of the following histologic subtypes of rhabdomyosarcoma has the worst
prognosis?

A. Alveolar
B. Embryonal
C. Spindle cell
D. Botryoid

Correct answer is A. RATIONALE: Data from the literature showed that histologic subtype
bears prognostic significance for rhabdomyosarcoma. Of these, alveolar histology has the
worst survival.

77. Which of the following represents an advantage of hyperfractionation?

A. Reduced acute toxicity


B. Reduced late effects to normal tissue
C. Decreased overall treatment time
D. Increased patient convenience

Correct answer is B. RATIONALE: For a given prescribed overall dose, decreasing the fraction
size would result in LENGTHENED overall treatment times. The difference in monitor units
needed to deliver each fraction would be overshadowed by the increased number of visits
required of the patient. With a smaller fraction size, the probability of late complications may be
lower, but the management would remain the same.

22 American College of Radiology


78. The majority of x-ray production in a typical diagnostic x-ray tube operating at 60 kVp
results from:

A. the Compton effect.


B. the photoelectric effect.
C. bremsstrahlung radiation.
D. characteristic x-rays from the tungsten target.

Correct answer is C. RATIONALE: This item requires differentiation of x-ray production


mechanisms.

79. Prior to the advent of chemotherapy, the overall survival rate for patients with
osteosarcoma treated with surgery alone was:

A. 05%.
B. 20%.
C. 50%.
D. 70%.

Correct answer is B. RATIONALE: Surgical ablation including amputation alone resulted in


long-term survival in approximately 20% of patients.

80. Which of the following statements about LCIS is true?

A. The median age of diagnosis is 50 to 60 years.


B. Architectural distortion is the most common finding on mammography.
C. It is associated with diffuse and bilateral proliferative activity in the breast tissue.
D. A lobular histology is associated with subsequent development of invasive cancer in
75% of patients.

THIS ITEM WAS NOT SCORED. RATIONALE: LCIS is not associated with any gross clinical or
mammographic features. It may present in the setting of fibrocystic changes. Typically, LCIS is
an incidental finding when biopsy of the breast tissue is completed for another reason. Median
age is 40 to 50 years and is seen more commonly in premenopausal women than in
postmenopausal women. Most invasive cancers seen in association with LCIS are ductal. Only
one third to one half of cases may present with invasive lobular histology. Reference: Harris, J,
et al. Diseases of the Breast. 3rd edition. 2004; pp 497-505.

Radiation Oncology In-Training Exam 2007 23


81. Which of the following factors has contributed most to the decline in incidence of distal
gastric cancers in the United States?

A. Use of proton pump inhibitors


B. Improved food storage and preservation
C. Improved detection of Helicobacter pylori
D. Routine upper endoscopy screening of patients at 50 years of age

Correct answer is B. RATIONALE: Epidemiologic studies suggest that the routine use of
refrigeration and the consequent avoidance of smoking and salting for food preservation is the
major reason behind the decline in gastric cancer in the United States.

82. Which of the following sequences of items in a beam path would be most appropriate for
delivery of 6-MeV electron-beam therapy?

A. Scattering foil, ionization chambers


B. High Z target, scattering foil, ionization chambers
C. High Z target, flattening filter, ionization chambers
D. Flattening filter, scattering foil, ionization chambers

Correct answer is A. RATIONALE: Requires knowledge of the linac's production of x-rays and
electrons.

83. Which of the following radiation doses should NOT be exceeded when stereotactic
radiosurgery is administered to the optic chiasm?

A. 04 Gy
B. 06 Gy
C. 08 Gy
D. 14 Gy

Correct answer is C. RATIONALE: A radiation dose of 8 Gy is the recommended upper limit


(14 Gy is too high).

84. Which of the following chemotherapy regimens is most appropriate in the treatment of
gastric cancer?

A. Cisplatin and VP-16


B. Gemcitabine and capecitabine
C. Epirubicin, cisplatin, and 5-FU
D. Cyclosphosphamide, methotrexate, and 5-FU

Correct answer is C. RATIONALE: Epirubicin, cisplatin, and 5-FU (ECF) comprise the
chemotherapy regimen used in the MAGIC trial, which showed a benefit to perioperative
chemotherapy, and in the ongoing Intergroup trial, which randomizes ECF to 5-FU/LV as given
in Intergroup 0116.

24 American College of Radiology


85. The use of an endorectal balloon and radiation therapy for prostate cancer has NOT
been shown to:

A. cause hypoxia.
B. reduce prostate motion.
C. improve high-dose rectal wall sparing.
D. be practical for target localization during daily treatment.

Correct answer is A. RATIONALE: An endorectal balloon has not been shown to cause
hypoxia in prostate cancer.

86. According to the GOG 88 (Stehman) trial for vulvar cancer, which of the following
outcomes is associated with patients who undergo definitive groin irradiation compared
to groin dissection alone, each in conjunction with a radical vulvectomy?

A. Worse local control and worse survival rates


B. Similar local control and similar survival rates
C. Improved local control and improved survival rates
D. Improved local control and similar survival rates

Correct answer is A. RATIONALE: In the GOG 88 trial, the irradiated patients had a
significantly worse local (inguinal) control and worse overall survival than patients treated with
surgery alone. This trial has been criticized, however, due to the fact that the groin irradiation
was prescribed only to 3 cm below the anterior skin surface significantly underdosing many
women's inguinal lymph nodes. Reference: Stehman, et al. International Journal of Radiation
Oncology, Biology, Physics. 1992;24:380-96.

87. Primary surgery is most appropriate for patients with which of the following types of
squamous cell carcinoma?

A. Stage T1N0 of the oropharyngeal portion of the posterior pharyngeal wall


B. Stage T1N0 of the floor of mouth
C. Stage T1N0 of the soft palate
D. Stage T2N0 of the base of tongue

Correct answer is B. RATIONALE: Primary surgery is the recommended therapy for most
tumors of the oral cavity, while radiation therapy is preferable to surgery for most tumors of the
oropharynx. Even experts who recommend primary surgery for oropharynx primaries would
agree that the added benefit from surgery is greatest for answer B.

88. What effect does external-beam radiation therapy have on lung function?

A. Increases TLC
B. Increases DLCO if chemotherapy is added
C. Decreases FEV1/VC ratio
D. Decreases DLCO

Correct answer is D. RATIONALE: Both radiation therapy alone and in combination with
chemotherapy cause an immediate decline in DLCO (corrected or uncorrected) starting within
1 month after radiation therapy with a nadir at 9 months.

Radiation Oncology In-Training Exam 2007 25


89. Approximately what percent of patients who have follicular non-Hodgkin lymphomas
present with B symptoms?

A. 75%
B. 50%
C. 25%
D. 05%

Correct answer is D. RATIONALE: B symptoms occur very infrequently in non-Hodgkin


lymphoma and usually are associated with stage III or IV disease.

90. What percent of neuroblastomas exhibit N-myc amplification?

A. 05%
B. 25%
C. 50%
D. 75%

Correct answer is B. RATIONALE: Overall, 25% of neuroblastomas exhibit N-myc


amplification.

91. Which of the following statements about radiation-induced fibrosis is true?

A. It is usually irreversible.
B. It can be easily managed by surgical excision.
C. It can be effectively treated using hyperbaric oxygen.
D. The severity is reduced with concurrent chemotherapy.

Correct answer is A. RATIONALE: Radiation-induced fibrosis is generally irreversible. The


emphasis therefore is on prevention. Surgical manipulation of irradiated tissues should only be
performed by experienced operators.

92. For a 6-MV x-ray beam, what is the most appropriate specification for flatness in the
radial and transverse directions?

A. Within +/-1% over 50% of the maximum field size at a depth of 1.5 cm
B. Within +/-1% over 80% of the maximum field size at a depth of 10 cm
C. Within +/-3% over 50% of the maximum field size at a depth of 10 cm
D. Within +/-3% over 80% of the maximum field size at a depth of 10 cm

Correct answer is D. RATIONALE: Requires knowledge of the common definition of beam


flatness.

26 American College of Radiology


93. Median survival is defined as the:

A. most prevalent time to death in the sample population.


B. point in time a survival probability on a Kaplan-Meier curve is 50%.
C. median of the survival-time intervals.
D. mean of the survival-time intervals.

Correct answer is B. RATIONALE: The median survival time is the time at which half the
subjects have died and half are still alive. It is easy to derive the median survival time from the
survival curve by drawing a horizontal line at 50% survival and seeing where it crosses the
curve.

94. Which of the following statements about the biological characteristics of DCIS is true?

A. Positive estrogen receptors are found in 90% of cases.


B. The HER-2/neu gene is overexpressed in 25% of cases.
C. The BRCA1 or BRCA2 germline mutation is found in 5% of patients.
D. Low-grade lesions are histologically continuous (with ≤5-mm gaps).

THIS ITEM WAS NOT SCORED. RATIONALE: DCIS tumors are ER-positive in 70% of cases.
HER-2/neu is positive in 55% of cases. High-grade lesions tend to be histologically continuous.
It is the low-grade tumors that tend to be more multifocal/multicentric. Reference: Burstein, HJ,
et al. Ductal carcinoma in situ of the breast. New England Journal of Medicine (NEJM).
2004;350:1430-1441.

95. Which of the following molecular events is required during apoptosis?

A. Cell cycle checkpoint inhibition


B. Epidermal growth factor receptor (EGFR) inhibition
C. Caspase activation
D. p21 activation

Correct answer is C. RATIONALE: The biochemical definition of apoptosis includes the


activation of executioner caspases (e.g., caspase 3 or 7). The activated caspase then cleaves
other intracellular targets such as PARP. In contrast, it is quite possible to have apoptosis and
caspase activation in the absence of p53 or EGFR. In other words, cancer cells lacking p53 or
EGFR may still be susceptible to having caspase activated through other pathways, resulting in
apoptosis.

96. What is the stage of a 5-cm gastric tumor that extends into, but not through, the
muscularis propria with 16 of 23 positive lymph nodes?

A. I
B. II
C. III
D. IV

Correct answer is D. RATIONALE: In gastric cancer, involvement of >15 lymph nodes is


classified as N3, which corresponds to stage IV.

Radiation Oncology In-Training Exam 2007 27


97. Which of the following chemotherapy agents recently has been shown to be effective in
treating hormone-refractory prostate cancer?

A. Docetaxel
B. Gemcitabine
C. Oxaliplatin
D. Topotecan

Correct answer is A. RATIONALE: Docetaxel has been shown to be more effective than the
traditional chemotherapy (e.g., estramustine, mitoxantrone, and prednisone).

98. Which of the following radiation protection terms is defined correctly?

A. The radiation-weighting factor is a unit-less multiplier that corrects the absorbed


dose by a factor related to the RBE of the type of radiation.
B. The committed equivalent dose is the product of the absorbed dose and a
tissue-specific weighting factor.
C. The collective effective dose is the product of the average equivalent dose to a
population and the number of persons of reproductive age.
D. The NCRP is a committee appointed by Congress that is charged with enforcement
of radiation-protection standards in the workplace.

Correct answer is A. RATIONALE: The radiation-weighting factor (WR) is a unit-less multiplier


that corrects absorbed dose (in Gy) by a factor that takes into account the relative biological
effectiveness (RBE) of different types of radiation to which an individual may be exposed. After
the correction is made, “dose equivalent” can be expressed on a common scale using sieverts
(Sv) as the unit of measure. The radiation-weighting factors for different types of radiation are
determined by the ICRP, and although based on experimentally measured values of RBE, are
often averaged or subject to other “judgment calls.” The appropriate use of WR values is for
estimating the probability of stochastic late effects, such as carcinogenesis or mutagenesis,
following low-dose or low-dose-rate irradiation. Reference: Hall and Giaccia. Radiobiology for
the Radiologist. 6th edition. 2006; Chapter 15, pp 226-227.

99. For a 6-MV x-ray beam, what is the most appropriate specification for symmetry in the
radial and transverse directions?

A. Within +/-1% over 80% of the maximum field size at a depth of 1.5 cm
B. Within +/-1% over 50% of the maximum field size at a depth of 10 cm
C. Within +/-1% over 80% of the maximum field size at a depth of 10 cm
D. Within +/-2% over 80% of the maximum field size at a depth of 10 cm

Correct answer is D. RATIONALE: This item requires an understanding of an important linac


parameter — beam symmetry.

28 American College of Radiology


100. Which of the following tumor stages is associated with a patient who has vulvar cancer
arising on the labia minora with involvement of the lower urethra?

A. T1
B. T2
C. T3
D. T4

Correct answer is C. RATIONALE: According to the AJCC Cancer Staging Manual, vulvar
tumors involving the lower urethra and/or vagina and anus are classified as stage T3.
Upper-urethral involvement is classified as stage T4.

101. What two emission products of 131I are used to manage patients with papillary thyroid
carcinoma?

A. An electron for radiation therapy and an electron for diagnostic imaging


B. An electron for radiation therapy and a photon for diagnostic imaging
C. A photon for radiation therapy and an electron for diagnostic imaging
D. A photon for radiation therapy and a photon for diagnostic imaging

Correct answer is B. RATIONALE: Iodine-131 is able to ablate thyroid tissue and tumor cells
because it emits a relatively low-energy electron that deposits energy within a few millimeters.
Iodine-131 is used to image thyroid cancer because a second emission product is a
kilovoltage-range photon that exits the body.

102. Which of the following statements about staging for non-small cell carcinoma of the lung
is true?

A. CT scan provides approximately 95% sensitivity and specificity.


B. Screening tests have reduced the mortality rate for high-risk patients.
C. PET scanning has replaced mediastinoscopy for determining cancer staging.
D. Positive PET scan findings require pathological confirmation.

Correct answer is D. RATIONALE: Mediastinoscopy is still considered the gold standard for
staging.

103. What is a major criterion for the diagnosis of multiple myeloma?

A. Calcium level of >12 mg/dL


B. Hemoglobin level of 8.5 g/dL
C. Plasmacytoma on tissue biopsy
D. Lytic bone lesions on skeletal survey

Correct answer is C. RATIONALE: Major criteria are plasmacytoma on tissue biopsy, bone
marrow plasmacytoses of >30% plasma cells, and elevated monoclonal globulin spike on serum
electrophoresis above a defined level.

Radiation Oncology In-Training Exam 2007 29


104. Which of the following body sites is most commonly involved in patients with
rhabdomyosarcoma?

A. Thorax
B. Extremities
C. Head and neck
D. Genitourinary tract

Correct answer is C. RATIONALE: This item is based on epidemiological data. Approximately


40% of rhabdomyosarcomas occur in the head and neck region.

105. Which of the following characteristics is NOT associated with a stochastic radiation
effect?

A. The effect is "all or none."


B. An increase in dose will increase the severity of the effect.
C. The risk of the effect increases linearly with dose, with no dose threshold.
D. The risk of the effect is probabilistic and occurs by chance for a given individual.

Correct answer is B. RATIONALE: A stochastic radiation effect is one that does not exhibit a
dose threshold; that is, there is no dose below which there is not some probability of a negative
consequence resulting from exposure, with that probability increasing with increasing dose, but
that otherwise occurs by chance for a given individual. The effect itself however is “all or none,”
and it does not increase in severity with increasing dose. Radiation-induced cell killing,
mutagenesis, and carcinogenesis are stochastic effects. Radiation effects that show dose
thresholds and increase in severity with increasing dose are termed non-stochastic or
“deterministic.” Early and late effects in normal tissues following radiation therapy and
radiation-induced cataracts are examples of deterministic radiation effects. Reference: Hall and
Giaccia. Radiobiology for the Radiologist. 6th edition. 2006; Chapter 10, pp 135-136.

106. Which of the following processes best explains how a radiation beam is delivered during
segmented IMRT?

A. Only deliver radiation while the MLC is stopped.


B. Leave the radiation beam on during the entire delivery sequence.
C. Continuously move all primary collimators and MLC leaves.
D. Move all MLC leaves in one direction only.

Correct answer is A. RATIONALE: In segmental IMRT, the beam is "Off" when the MLC
leaves are moving to their positions. In dynamic IMRT, the beam is constantly "On" when the
leaves are moving.

107. Which of the following chemotherapy agents is most effective for desmoid tumors?

A. Vincristine
B. Vinblastine
C. Cyclophosphamide
D. Etoposide

THIS ITEM WAS NOT SCORED.

30 American College of Radiology


108. Which of the following statements about the management of brain metastases
secondary to breast cancer is true?

A. Prednisone therapy should be initiated with a loading dose of 20 mg, followed by


10 mg four times daily.
B. Symptomatic improvement is expected between 24 to 72 hours after initiation of
therapy.
C. Treatment is more effective for focal neurological deficits than for headaches.
D. Dexamethasone must be administered four times daily because of its half-life
properties.

Correct answer is B. RATIONALE: Corticosteroids are associated with multiple side effects
including myopathy, weight gain, fluid retention, hyperglycemia, insomnia, gastritis, and
immunosuppression. The frequency and severity of these symptoms can be modified by using
the lowest dose possible. Most patients are started on dexamethasone because of relatively
little corticosteroid, decreasing the risk of fluid retention and providing a lower risk of infection
and cognitive impairment. A loading dose of 10 mg is given with 4 mg qid. Dexamethasone is
given in 4 divided doses, but its biologic half-life could allow bid administration. References:
Harris, JR. Diseases of the Breast. 3rd edition. Lippincott, Williams and Wilkins, 2004;
pp 1207-1208. Vecht, CJ, et al (Wiley R, ed). Use of glucosteroids in neuron-oncology.
Neurological Complications of Cancer. 1995; New York. Marcel Dekker.

109. What percent of pituitary adenomas are secretory tumors?

A. 10%
B. 30%
C. 45%
D. 65%

Correct answer is D. RATIONALE: As many as 70% of pituitary adenomas are


endocrinologically active, and the presence of a hypersecretory state is the most common mode
of presentation.

110. What is the most common histology of tumors involving the terminal ileum?

A. Sarcoma
B. Carcinoid
C. Lymphoma
D. Adenocarcinoma

Correct answer is B. RATIONALE: Carcinoid is the most common histology of tumors involving
the terminal ileum.

Radiation Oncology In-Training Exam 2007 31


111. Which of the following statements about the randomized trial comparing
conventional-dose (70.2 Gy) versus high-dose (79.2 Gy) radiation therapy using a
combination of conformal photon and proton beams for clinically localized prostate
cancer is true?

A. High-dose radiation therapy does not improve biochemical outcome.


B. The advantage of high-dose radiation therapy is observed in both high-risk and
low-risk patient groups.
C. There is increased acute or late rectal toxicity in the high-dose group.
D. There is increased acute or late urinary toxicity in the high-dose group.

Correct answer is B. RATIONALE: The advantage is seen in both low- and high-risk patients.

112. According to GOG 37 (Homesley), the 2-year survival rate of patients who have vulvar
cancer with positive inguinal lymph nodes and undergo adjuvant pelvic-inguinal
irradiation compared to pelvic lymphadenectomy alone is:

A. 44% versus 50%.


B. 54% versus 68%.
C. 68% versus 54%.
D. 73% versus 72%.

Correct answer is C. RATIONALE: In GOG 37, patients with vulvar cancer and positive groin
lymph nodes after radical vulvectomy and bilateral groin dissection were randomized to
pelvic-inguinal irradiation or pelvic lymphadenectomy. Patients undergoing irradiation had a
significantly improved survival compared to patients undergoing surgery alone (2-year survival
of 68% vs. 54%, p = 0.03). Reference: Homesley, et al. Obstetrics Gynecology.
1986;68:733-40.

113. Which of the following pathological stages of breast cancer is associated with a patient
who has a 5.0-cm primary tumor and 12/15 positive, non-fixed lymph nodes?

A. pT3N2
B. pT3N3
C. pT2N1
D. pT2N3

Correct answer is D. RATIONALE: Pathologic tumor size: Stage T2 tumor is more than 2 cm
but not more than 5 cm in greatest dimension. Stage T3 tumor is more than 5 cm in greatest
dimension. Lymph nodal stage: Stage N1 involves metastases to the ipsilateral axillary lymph
nodes, 1 to 3 positive and/or IM nodes with microscopic disease detected by sentinel lymph
node dissection but not clinically apparent via CT or PE. Stage N2 involves metastases to 4 to
9 axillary lymph nodes (at least one tumor deposit greater than 2.0 mm). Stage N3 involves
metastases to 10 or more axillary lymph nodes or infraclavicular lymph nodes or clinically
apparent ipsilateral IM nodes with 1 or more positive axillary lymph nodes or more than
3 axillary lymph nodes with microscopic metastases in IM nodes or positive ipsilateral
supraclavicular lymph nodes. Reference: AJCC Cancer Staging Manual, (6th edition), 2002;
American Joint Committee on Cancer, Springer Press, pp 227-228.

32 American College of Radiology


114. What stage of non-small cell lung cancer is most appropriate for a patient who has a
3-cm mass in the right upper lobe, mediastinal adenopathy, and a 2-cm nodule in the
right lower lobe of the lung?

A. Stage IIB
B. Stage IIIA
C. Stage IIIB
D. Stage IV

Correct answer is D. RATIONALE: AJCC Cancer Staging Manual.

115. Which of the following radiation dose ranges is most commonly used to treat solitary
plasmacytoma?

A. 15 Gy to 20 Gy
B. 24 Gy to 30 Gy
C. 40 Gy to 50 Gy
D. 60 Gy to 70 Gy

Correct answer is C. RATIONALE: Dose response data indicate that 40 Gy to 50 Gy will


control most solitary plasmacytomas.

116. What is the most common group classification of rhabdomyosarcoma at initial


diagnosis?

A. Group I
B. Group II
C. Group III
D. Group IV

Correct answer is C. RATIONALE: This item is based on epidemiological data. The most
common presentation is Group III disease. Reference: Donaldson, SS, and Anderson, J.
Factors that influence treatment decisions in childhood rhabdomyosarcoma. Intergroup
Rhabdomyosarcoma Study Group of the Children's Cancer Group, the Pediatric Oncology
Group, and the Intergroup Rhabdomyosarcoma Study Group Statistical Center. Radiology.
1997;203:17-22.

117. Which of the following best describes the LENT/SOMA scoring system?

A. It predicts the probability of late effects.


B. It predicts tumor response to chemoradiation.
C. It standardizes assessments of the severity of acute effects.
D. It standardizes assessments of the severity of late effects.

Correct answer is D. RATIONALE: The Late Effects Normal Tissue (LENT) Task Force -
Subjective, Objective, Management, Analytic (SOMA) scale is intended to standardize the
assessments of the incidence and severity of late radiation effects.

Radiation Oncology In-Training Exam 2007 33


118. Which of the following disadvantages is associated with intensity-modulated radiation
therapy (IMRT) versus conformal radiation therapy?

A. Decreased leakage radiation to the entire body of the patient


B. Decreased absorbed dose to normal tissues and critical structures
C. Decreased monitor unit to dose efficiency
D. Decreased "beam-on" times per field

Correct answer is C. RATIONALE: Intensity-modulated radiation therapy (IMRT) generally


requires a large number of monitor units for the amount of absorbed dose delivered.

119. Which of the following lesions is most likely to be associated with otalgia?

A. Stage T1N1 of the base of tongue


B. Stage T1N1 of the posterior pharyngeal wall of the hypopharynx
C. Stage T2N0 of the middle section of the hard palate
D. Stage T2N1 of the true vocal cord

Correct answer is A. RATIONALE: Earache is a common symptom at diagnosis of carcinoma


of the base of tongue or supraglottic larynx. The mechanism for ear pain in this setting is
referred pain along the nerves of Jacobson and Arnold.

120. Which of the following statements associated with the postmastectomy Danish trials
investigating the use of radiation therapy is true?

A. The trials studied the use of anthracycline chemotherapy for all premenopausal
patients.
B. The trials studied the use of tamoxifen over a 5-year period for all postmenopausal
patients.
C. The trials found that the favorable association of radiation therapy with improved
10-year overall survival was lost after 15 years of follow-up.
D. The trials found an association between the use of radiation therapy and reduced
rates of subsequent development of metastatic disease.

Correct answer is D. RATIONALE: Postoperative radiation therapy in high-risk


postmenopausal breast cancer patients were given adjuvant tamoxifen. The Danish trial was
completed with CMF chemotherapy. Tamoxifen is administered to postmenopausal patients for
1 year. The 15-year analysis has upheld the 10-year findings:
Disease-free survival Overall survival P
Tamoxifen alone 24% 36% <.001
Tamoxifen + XRT 36% 45% .03
Reference: Overgaard, M, et al: Lancet. 1999;353:1641-1648.

121. Which of the following pathologic subtypes is most commonly associated with diffuse
astrocytoma in adults?

A. Pilocytic
B. Fibrillary
C. Protoplasmic
D. Gemistocytic

Correct answer is B. RATIONALE: Fibrillary astrocytoma is the most common subtype.

34 American College of Radiology


122. Which of the following statements about the absolute and relative risk models of
radiation carcinogenesis is true?

A. The absolute risk model is best described by the excess incidence of


radiation-induced solid tumor in Japanese A-bomb survivors.
B. The relative risk model is best described by the excess incidence of
radiation-induced leukemia in Japanese A-bomb survivors.
C. The absolute risk model predicts that radiation exposure induces a discrete "crop" of
excess cancers that, after the appropriate latency period, are added to the
spontaneous incidence of that type of cancer.
D. The relative risk model predicts that radiation exposure induces a discrete "crop" of
excess cancers that, after the appropriate latency period, are added to the
spontaneous incidence of that type of cancer.

Correct answer is C. RATIONALE: The absolute risk model of radiation carcinogenesis


assumes that radiation exposure of a population induces a discrete “crop” of excess cancers
over and above the natural cancer incidence and unrelated to it. After the appropriate latency
period, these excess cancers appear and are added to the natural incidence, but thereafter, the
cancer incidence returns to baseline. On the other hand, the relative risk model predicts that
radiation exposure of a population causes a multiplicative increase in the natural cancer
incidence for each cancer type and at all ages, and because cancer tends to be a disease of
older age in general, most of the radiation-induced excess cancers would be expected later in
the lives of the irradiated cohort. For the Japanese A-bomb survivors, most of whom are now at
or approaching old age (when the greatest number of radiation-induced solid tumors are to be
expected), a modified version of the relative risk model is favored by the BEIR committee. The
time-dependent relative risk model assumes that the radiation dose, the dose squared, age at
exposure, time elapsed since exposure, and in some cases, gender, all contribute to the excess
cancer risk. Reference: Hall and Giaccia. Radiobiology for the Radiologist. 6th edition. 2006;
Chapter 10, pp 138-139.)

123. Which radiation source is governed by the Nuclear Regulatory Commission (NRC)?

A. Cobalt
B. Radium
C. Linear accelerators
D. Therapeutic x-ray machines

Correct answer is A. RATIONALE: The NRC only regulates byproduct material, such as
cobalt. Uranium is naturally occurring, and accelerators and x-ray machines are not considered
materials.

124. Which of the following types of cancer most commonly occurs in the GI tract in the U.S.
population?

A. Colon
B. Rectal
C. Esophageal
D. Gastric

Correct answer is A. RATIONALE: There are approximately 150,000 cases of colon cancer
per year, significantly more than the other choices. Reference: CA: A Cancer Journal for
Clinicians. 2006;56:106-130.

Radiation Oncology In-Training Exam 2007 35


125. Which of the following statements about brachytherapy for prostate cancer is true?

A. Patient outcome after brachytherapy alone has not been shown to be equivalent to
radical prostatectomy and external-beam radiation therapy in patients with localized
prostate cancer.
B. Brachytherapy alone is best for patients who have stage T1 to stage T2a prostate
cancer with a Gleason score of ≤8 and a prostate-specific antigen (PSA) level of
<10 ng/mL.
C. Differences in half-life and dose rate suggest that 125I is preferred for slow-growing,
low-grade tumors, and 103Pd is preferred for high-grade tumors.
D. The addition of androgen deprivation to external-beam radiation therapy is not
useful in patients with high-risk factors.

Correct answer is C. RATIONALE: Although there are no clinical data demonstrating an


advantage, differences in half-life and dose rate suggest that I-125 is better suited for
slow-growing, low-grade tumors; whereas Pd-103 is better for high-grade disease.

126. Where is vulvar carcinoma most commonly located?

A. Clitoris
B. Mons pubis
C. Vaginal vestibule
D. Labia

Correct answer is D. RATIONALE: Most vulvar cancers arise on the labia majora or minora.
The second most common site is the clitoris. The mons pubis and vaginal vestibule are
uncommon sites.

127. A 35-year-old woman who has follicular thyroid carcinoma undergoes thyroidectomy and
neck dissection, which reveals positive margins and lymph nodes. Which of the
following radiation therapy plans is most appropriate?

A. A 150-mCi dose of 131I, followed by external-beam radiation therapy to a dose of


50 Gy at 2 Gy per fraction
B. External-beam radiation therapy to a dose of 50 Gy at 2 Gy per fraction, followed by
a 150-mCi dose of 131I
C. External-beam radiation therapy to a dose of 70 Gy at 2 Gy per fraction only
D. A 150-mCi dose of 131I only

Correct answer is D. RATIONALE: In a young adult, the only conceivable indication for
external-beam radiation therapy is recurrent disease following I-131 therapy. This patient does
not have an indication for external-beam radiation therapy.

36 American College of Radiology


128. The risk estimate for excess cancer mortality due to chronic, low-level radiation
exposure is approximately 4% per sievert (Sv) for the working population, compared to
5% per Sv for the general population. What is the most likely explanation for this
difference?

A. Working adults exhibit longer latency periods for radiation-induced cancers.


B. The adult working population contains fewer elderly persons who demonstrate the
greatest excess of radiation-induced cancers.
C. Children in the general population are more sensitive to radiation-induced cancer.
D. The incidence of radiation-induced cancer among children in the general population
follows the relative risk model, which predicts a larger number of excess cancers
than the absolute risk model.

Correct answer is C. RATIONALE: For the general population, the ICRP recommends slightly
higher risk estimates than for the adult working population because of the increased
susceptibility of the young.

129. Which of the following personnel must be present during a high-dose-rate (HDR)
procedure after the procedure begins?

A. Only an authorized physician


B. Only a qualified medical physicist
C. Both an authorized physician and a qualified medical physicist
D. Both the licensee and an authorized user

Correct answer is C. RATIONALE: Once an HDR treatment begins, both an authorized


physician and an authorized medical physicist, both trained in HDR emergency procedures,
must be present for the entire treatment.

130. What is the best treatment for malignant mesothelioma?

A. Extrapleural pneumonectomy, followed by chemotherapy and adjuvant radiation


therapy to a dose of 50 Gy to 60 Gy
B. Pleurodesis, followed by systemic chemotherapy with subsequent external-beam
radiation therapy to a dose of 60 Gy in 20 fractions
C. IMRT to a localized positive margin to a radiation dose of 70 Gy
D. IMRT to the entire lung to a radiation dose of 45 Gy

Correct answer is A. RATIONALE: For healthy patients with malignant mesothelioma, the best
results have been achieved with trimodality therapy.

131. Which of the following factors is most strongly linked to an increased risk for the
development of multiple myeloma?

A. Alcohol use
B. Tobacco use
C. Mustard gas exposure
D. Ionizing radiation exposure

Correct answer is D. RATIONALE: Ionizing radiation exposure is a risk factor for multiple
myeloma, as shown by a marked increase of this tumor in Hiroshima survivors.

Radiation Oncology In-Training Exam 2007 37


132. Which of the following types of acute leukemia occurs most commonly during childhood?

A. Pre-B cell lymphocytic


B. Mature B-cell lymphocytic
C. Mature T-cell lymphocytic
D. Monocytic myelogenous

Correct answer is A. RATIONALE: This item is based on the epidemiology of childhood


leukemia. Acute lymphocytic leukemia (ALL) comprises 80% of all acute leukemias in
childhood. Most are the pre-B cell type.

133. Which of the following cancer genotypes is NOT correctly matched with the resulting
phenotype?

A. Down-regulation of the Bcl-2 gene → evasion of apoptosis


B. Up-regulation of vascular endothelial growth factor (VEGF) gene → tumor
angiogenesis
C. Overexpression of the ras oncogene → growth in the absence of "go" signals
D. Activation of telomerase → cellular immortality

Correct answer is A. RATIONALE: The Bcl-2 gene codes for an anti-apoptosis protein, and as
such, will allow cells to evade apoptosis when up-regulated, not down-regulated. Reference:
Gibbs, WW. Scientific American. 2003;289: 56-65.

134. How many mrem equal 10 mSv?

A. 001 mrem
B. 0010 mrem
C. 0100 mrem
D. 1000 mrem

Correct answer is D. RATIONALE: There is a direct conversion from conventional units (mrem)
to SI units (mSv).

135. Which of the following statements about postoperative brachytherapy for fibrosarcoma of
an extremity is true?

A. The entire muscle compartment including the tumor bed should be irradiated.
B. Catheters are loaded with radioactive sources approximately 5 days after surgery.
C. Brachytherapy gives better local control than external-beam radiation therapy.
D. Brachytherapy is effective for both low-grade and high-grade sarcomas.

Correct answer is B. RATIONALE: Catheters are loaded 5 days after surgery to prevent
wound-healing complications.

38 American College of Radiology


136. Which of the following results is associated with concurrent chemoradiation therapy
versus radiation therapy alone for patients with locally advanced laryngeal cancer,
based on the RTOG 91-11 study?

A. Same rate of distant metastases


B. Improved swallowing function
C. Improved overall survival
D. Improved local control

Correct answer is D. RATIONALE: Based on results of the RTOG 91-11 study.

137. Which of the following stages of colon cancer is most likely to benefit from adjuvant
radiation therapy?

A. Stage T4N0 tumor of the sigmoid colon with invasion into the pelvic sidewall
B. Stage T3N2 tumor of the transverse colon
C. Stage T2N1 tumor of the descending colon
D. Stage T2N0 tumor of the appendix

Correct answer is A. RATIONALE: Although the only randomized trial of radiation therapy vs.
no radiation therapy for colon cancer did not demonstrate an advantage to treatment, several
retrospective studies and first principles of radiation suggest a T4 tumor with clear invasion into
adjacent organs would be the most likely of the above choices to benefit from adjuvant therapy.

138. Which of the following statements about receptor/ligand interactions is most accurate?

A. They usually occur first at the nucleus.


B. Their effect is mediated by down-stream pathways.
C. The ligand is an antibody that targets the receptor with specificity.
D. Mutation of the ligand causes resistance to receptor-targeted therapy.

Correct answer is B. RATIONALE: Ligands can be antibodies, hormones, growth factors, or


synthetically designed small molecules. The initial receptor/ ligand interaction is at the cell
surface. Mutations in the receptor may lead to altered signaling (e.g., through reduced ligand
binding or the ability to signal to downstream pathways even in the absence of a ligand).

139. Body monitors must be provided to radiation oncology personnel who:

A. work in the clinic.


B. might receive at least 50 mR/year.
C. might receive at least 100 mR/year.
D. might receive at least 250 mR/year.

THIS ITEM WAS NOT SCORED. RATIONALE: Regulations require monitoring of anyone who
is at risk of receiving at least 10% of the annual occupational exposure limit, which is
5000 mRem.

Radiation Oncology In-Training Exam 2007 39


140. Which of the following side effects is NOT associated with long-term hormonal therapy
combined with radiation therapy for prostate cancer?

A. Decreased sexual libido and hot flashes


B. Osteoporosis and muscle weakness
C. Increased late GU toxicity
D. Increased late GI toxicity

Correct answer is C. RATIONALE: Options A and B are obvious side effects of long-term
hormonal uses. In addition, RTOG 92-02 showed there was a small but significantly increased
incidence of grade 3, 4, and 5 late GI toxicities. Reference: Journal of Clinical Oncology. 2003
(Nov 1);21(21):3972-8.

141. What is the second most common histology associated with vulvar cancers?

A. Adenocarcinoma
B. Adenosquamous carcinoma
C. Squamous cell carcinoma
D. Melanoma

Correct answer is D. RATIONALE: The most common tumor histology for vulvar cancers is
squamous cell (85%). The second most common histology is melanoma, comprising about
10% of all cases. Other histologies are significantly less common (<5%).

142. What is the most common genetic abnormality associated with low-grade gliomas?

A. 1p
B. 10q
C. 17p
D. 19q

Correct answer is C. RATIONALE: Deletion of chromosome 17p contains the p53 gene.

143. Which of the following processes is associated with the loss of heterozygosity?

A. Activation of oncogenes
B. Inactivation of tumor suppressor genes
C. Hypermethylation of histone proteins
D. Initiation of apoptosis

Correct answer is B. RATIONALE: Gene conversion secondary to a loss of heterozygosity is a


recognized mechanism for the inactivation of tumor suppressor genes.

40 American College of Radiology


144. Which of the following materials would be best suited for neutron shielding?

Walls Door
A. Iron Lead
B. Steel Concrete
C. Lead Borated polyethylene
D. Concrete Borated polyethylene

Correct answer is D. RATIONALE: Neutrons are best absorbed by materials that are either
hydrogenous (concrete) or have a larger nuclear cross section (boron).

145. Which of the following statements about asbestos exposure and mesothelioma is true?

A. The carcinogenic effects appear to result from its physical properties rather than its
chemical structure.
B. Mesothelioma cell lines appear to be less sensitive than non-small cell lung cancer
cell lines to radiation.
C. Screening of asbestos workers for mesothelioma is an effective way to improve the
cure rate of this disease.
D. It is rare for mesothelioma to be misdiagnosed.

Correct answer is A. Reference: Devita, VT, Hellman, S, Rosenberg, SA. Cancer Principles
and Practice of Oncology. 6th edition. Chapter 40, Sec. 2.

146. Which of the following anatomical sites is most commonly associated with extraosseous,
soft tissue plasmacytomas?

A. Back
B. Pelvis
C. Lower extremity
D. Head and neck

Correct answer is D. RATIONALE: Extramedullary plasmacytomas arise most frequently in the


head and neck and upper respiratory tract.

147. Which of the following types of acute leukemia is most likely to be associated with a
12-year-old boy who presents with a large mediastinal mass and lymphoblasts in his
blood?

A. Pre-B cell lymphocytic


B. Mature B-cell lymphocytic
C. Mature T-cell lymphocytic
D. Monocytic myelogenous

Correct answer is C. RATIONALE: This item is based on the epidemiology of childhood


leukemia. This is a classic presentation of mature T-cell lymphocytic leukemia (older boy with a
mediastinal mass).

Radiation Oncology In-Training Exam 2007 41


148. Which of the following statements about gene expression profiling of tumor cells is true?

A. Fewer control samples are required to confirm the results.


B. RT-PCR or Northern blotting is not required to confirm the results.
C. It is performed by assessing the binding of total RNA from tumor cells to microchips
containing an array of probes.
D. It is performed by comparing DNA sequences of normal cells with tumor cells.

Correct answer is C. RATIONALE: Expression profiling is generally performed in core facilities


or specialized labs, but it always requires confirmation by reverse-transcriptase, or realtime
PCR, or Northern blotting to validate that the mRNA levels of specific genes are truly increased
or decreased.

149. Which of the following errors would have the most serious consequences for a target
dose to a 10-cm-wide volume treated with an AP field?

A. Incorrect geometric lateral shift of 4 cm


B. Incorrect photon energy (6 MV vs. 18 MV) prescribed to 10-cm depth
C. Miscalculation of monitor units by 15%
D. Missing a 15° wedge

Correct answer is A. RATIONALE: A missing 15° wedge would create approximately a 20%
error, and the incorrect energy would create an error of approximately 15%.

150. Which of the following cell markers is most commonly associated with classic Hodgkin
lymphoma?

A. CD15+, CD30+
B. CD15+, CD30-
C. CD15-, CD30+
D. CD15-, CD30-

Correct answer is A. RATIONALE: Classical Hodgkin lymphoma has cell markers CD15+,
CD30+, and can be CD20+ or CD20-. A is the only possible answer.

42 American College of Radiology


151. Which of the following statements about brain metastases secondary to breast cancer is
true?

A. Breast cancer is the most common cause of brain metastases in women.


B. HER-2/neu gene overexpression is not associated with an increased risk of brain
metastases.
C. Increased tumor size and number of positive lymph nodes are associated with an
increased risk of brain metastases.
D. Brain metastases are most likely to involve a solitary site.

THIS ITEM WAS PARTIALLY SCORED (BOTH OPTIONS C & D SCORED CORRECT).
RATIONALE: The most common cause of brain metastases in women is lung cancer. Breast
cancer is the second leading cause. In patients who have breast cancer and are screened for
enrollment in clinical trials of systemic therapy, 14.8% of patients had clinically occult CNS
metastases. Of these patients, tumors with HER-2/neu gene overexpression and the number of
metastatic sites were significant predictors for CNS involvement by multivariate analysis. Breast
cancer rarely presents with brain metastases before detection of the primary cancer. Brain
metastases are more likely to present in premenopausal women with aggressive, widely
metastatic disease. There is no relationship among the size, site of the primary tumor or the
number of positive lymph nodes, and the subsequent development of brain metastases.
ER-negative tumors are more likely to metastasize to the brain than are ER-positive tumors.
References: Harris, JR. Diseases of the Breast, 3rd edition, Lippincott, Williams and Wilkins,
2004; pp 1205-1218. Miller, K, et al. Occult central nervous system involvement in patients with
metastatic breast cancer: prevalence, predictive factors and impact on overall survival. Annals
of Oncology. 2003;4:1972-1977. Sparrow, GE, et al. Brain metastases from breast cancer:
clinical course, prognosis and influence of treatment. Clinical Oncology. 1981;7:291-301.
Stewart, JF, et al. Estrogen receptors, sites of metastatic disease and survival in recurrent
breast cancer. European Journal of Cancer. 1981;17:449-453.

152. Which of the following types of astrocytoma is associated with tuberous sclerosis?

A. Fibrillary
B. Gemistocytic
C. Protoplasmic
D. Subependymal giant cell

Correct answer is D. RATIONALE: Subependymal giant cell astrocytomas occur in 5% of


patients with tuberous sclerosis.

Radiation Oncology In-Training Exam 2007 43


153. Which of the following factors is most likely responsible for the development of cancer,
given the very low frequency of spontaneous mutations characteristic of human cells?

A. Up-regulation of telomerase
B. Decreased immunosurveillance at the tumor site
C. Hypermutability of cells during the early stages of carcinogenesis
D. Alternate splicing of oncogenes and tumor suppressor genes during transcription

Correct answer is C. RATIONALE: An average of 3 to 7 different gene mutations is necessary


to convert a normal cell into a cancerous one capable of unlimited proliferation and invasive
and/or metastatic behavior. However, assuming that the spontaneous mutation rate of a normal
human cell is in the range of 10-6 mutations per gene per cell division, that most cells in the adult
body don’t divide, and that even those that do have a limited replicative lifespan (for the most
part), it would seem highly unlikely that all of these required mutations would ever accumulate in
a single cell over the organism’s lifetime. Nevertheless, approximately 40% of people are
diagnosed with cancer during their lifetimes, and some 20% to 25% of patients die from their
disease. The most prevalent explanation for this apparent paradox is that cells that acquire one
or two of the prerequisite carcinogenic mutations become hypermutable and can more readily
accumulate the additional mutations that allow them to progress to full malignancy in relatively
short order. This hypermutability is usually referred to as “genomic instability,” and it is thought
to occur secondary to the inactivation of tumor suppressor genes, the activation of oncogenes,
or both. What remains to be seen is whether these genetic changes are themselves the cause
of genomic instability or early indicators of it; that is, whether the actual initiating mutation(s) are
instead in as yet unidentified “master genes” that supervise DNA replication and repair, or
coordinated cell division. (Epigenetic changes, e.g., post-translational modifications of
chromatin that alter its structure or activity, and therefore, gene expression, can also contribute
to genomic instability.) References: Tannock, Hill, Bristow, and Harrington. The Basic Science
of Oncology, 4th edition, 2005; Chapter 5. Alberts, Bray, Lewis et al. Molecular Biology of the
Cell, 3rd edition. 1994; Chapter 24, pp 1255-1262. Gibbs, WW. Untangling the roots of cancer.
Scientific American. 2003;289: 56-65.

154. What is the maximum transmission allowed through collimating jaws?

A. 0.1%
B. 0.5%
C. 1.0%
D. 2.0%

Correct answer is B. RATIONALE: The maximum jaw transmission allowed is 0.5%, which is
specified in many regulations.

155. Approximately what percent of colorectal cancers are due to hereditary nonpolyposis
colorectal cancer (HNPCC)?

A. 01%
B. 05%
C. 15%
D. 25%

Correct answer is B. RATIONALE: It is suggested that "up to 6%" of colorectal cancers are
due to hereditary nonpolyposis colorectal cancer (HNPCC). Reference: Gastroenterology. 1993
(May);104(5):1535-49.

44 American College of Radiology


156. What percent of patients diagnosed with testicular seminoma have elevated
α-fetoprotein serum levels?

A. 00%
B. 10%
C. 20%
D. 30%

Correct answer is A. RATIONALE: An elevation of serum α-fetoprotein (AFP) is diagnostic for


testicular non-seminoma.

157. Which of the following types of gynecologic cancer has a similar pattern of tumor spread
as that associated with fallopian tube cancer?

A. Vulvar
B. Ovarian
C. Endometrial
D. Cervical

Correct answer is B. RATIONALE: The primary pattern of spread of fallopian tube cancers
closely resembles that seen in cancers of the ovary, with local extension to adjacent pelvic
structures and involvement of the peritoneum and omentum.

158. Which of the following genetic diseases characterized by radiation sensitivity and/or
increased cancer predisposition is correctly matched to the gene mutation and molecular
defect?

A. Li-Fraumeni syndrome → germline mutation in the RB gene (cell cycle regulator)


B. Nijmegen breakage syndrome → mutation in the MRE11 gene (DNA damage
response)
C. Lynch syndrome → mutation in the hMSH2 and/or hMLH1 genes (mismatch repair
components)
D. Cockayne syndrome → mutation in the BLM gene (DNA helicase)

Correct answer is C. RATIONALE: Li-Fraumeni syndrome is caused by a germline mutation in


the TP53 gene, not the RB gene. Nijmegen breakage syndrome is caused by a mutation in the
NBS1 gene, not the MRE11 gene. Cockayne syndrome is caused by a mutation in the CS
gene, not the BLM gene.

159. Electrons lose energy in water by approximately how many million electron volts (MeV)
per centimeter?

A. 0.5
B. 1.0
C. 2.0
D. 5.0

Correct answer is C. RATIONALE: By mass stopping power ratios, it can be derived that
electrons lose approximately 2 MeV per centimeter.

Radiation Oncology In-Training Exam 2007 45


160. What is the probability of being within two standard deviations of the mean for data with
an exactly normal distribution?

A. 50%
B. 68%
C. 95%
D. 99%

Correct answer is C. RATIONALE: The probability of being within 2 SD of the mean is 95%.

161. Which of the following statements about malignant mesothelioma is true?

A. The median patient survival time is 24 months.


B. Most patients have bilateral disease at presentation.
C. Diagnosis is most commonly established via cytology of pleural effusions.
D. It commonly spreads along the biopsy tract.

Correct answer is D. RATIONALE: It is important to know where malignant mesothelioma


spreads since these tracts have to be covered by a radiation port.

162. Which of the following factors is associated with a higher risk for the development of
Hodgkin lymphoma in economically developed countries?

A. Early birth order


B. Increased number of siblings
C. Increased number of playmates as child
D. Chronic immunosuppressant treatment

Correct answer is A. RATIONALE: Hodgkin lymphoma has been found to be related to the
level of maternal education for younger patients, decreased number of playmates, decreased
number of siblings, and early birth order in economically developed countries.

163. Which of the following factors is most important in establishing a favorable prognosis for
a child with posterior fossa ependymoma?

A. Excellent response to induction chemotherapy


B. Achievement of a gross total resection
C. Absence of p53 gene deletion
D. Patient age >8 years

Correct answer is B. RATIONALE: The extent of surgical resection has been the single most
important prognostic factor in patients over 18 months of age. (St. Jude, CCG).

46 American College of Radiology


164. Clinical radiation sensitivity is consistently associated with the absence of:

A. p53.
B. HER-2/neu.
C. gamma-H2AX.
D. ATM.

Correct answer is D. RATIONALE: While phosphorylation of H2AX (i.e. "gamma-H2AX") is


often considered a marker of the DNA damage response, humans deficient for H2AX have not
been described. Patients deficient for p53 (e.g. Li-Fraumeni syndrome) are prone to increased
malignancies, but have not been reported to show unusual toxicities after radiation therapy.

165. What is the percent dose at the surface for 20-MeV electrons?

A. 25%
B. 40%
C. 90%
D. 99%

Correct answer is C. RATIONALE: Electrons have a much higher surface dose than photons
and increase with increasing energy. A proper rule of thumb for electron percent depth dose
(PDD) is energy divided by 4 is approximately equal to the 90% depth.

166. Which of the following statements about osteosarcoma is true?

A. It rarely occurs in African Americans.


B. The amount of tumor necrosis at resection is an important prognostic factor.
C. The addition of ifosfamide and etoposide to standard VACA-based chemotherapy
improves survival for patients with nonmetastatic disease.
D. Radiation therapy is the preferred treatment for local control.

Correct answer is B. RATIONALE: The amount of tumor necrosis at resection is a prognostic


indicator and surgery is the local treatment of choice. Options A and C occur in Ewing sarcoma.

167. Which of the following statements about leptomeningeal metastases secondary to breast
cancer is true?

A. A headache is the most common symptom in patients at presentation.


B. A contrast-enhanced CT scan is best for establishing the diagnosis.
C. Radiation therapy should be administered to the entire CNS axis.
D. The median survival rate is 4 months, but longer survival times are associated with
breast cancer.

Correct answer is D. RATIONALE: Spinal symptoms are the most common presentation of
leptomeningeal disease. Limb weakness with paresthesias and pain occur in the affected limb.
Headaches are the most common symptom of cerebral disease. Diplopia is the most common
cranial nerve symptom. Gadolinium-enhanced MRI is the modality of choice for both cranial
and spinal sites. CT is less sensitive and is not useful for spinal imaging unless a contrast
agent is used in the subarachnoid space. Radiation therapy should be given to symptomatic
areas and to sites of bulky disease. Radiation is the best treatment for focal leptomeningeal
nodules. Focal radiation can help restore normal CNS flow and facilitate effectiveness of
intrathecal therapies. Reference: Harris, JR. Diseases of the Breast. 3rd edition. Lippincott,
Williams & Wilkins, 2004; pp 1231-1240.

Radiation Oncology In-Training Exam 2007 47


168. Oligodendrogliomas most commonly exhibit which of the following types of pathological
pattern?

A. Necrosis
B. Fried egg appearance
C. Rosenthal fibers
D. Globoid cells

Correct answer is B. RATIONALE: The “fried egg appearance” is secondary to prominent


nuclei surrounded by a cytoplasmic halo.

169. Which of the following microenvironmental or treatment-related conditions would inhibit


sublethal damage recovery?

A. Maintenance of cells at or below 5°C


B. Maintenance of cells at an oxygen concentration of 20 mm Hg
C. Maintenance of cells in a contact-inhibited, quiescent state
D. Preexposure of cells to an initial dose of 2 Gy

Correct answer is A. RATIONALE: Sublethal damage recovery is a fairly robust process that,
of the choices given, is only inhibited under conditions of extreme temperature (maintenance of
cells below about 5oC or above about 40oC) or severe hypoxia (oxygen concentration below
about 3 mm Hg). Repair occurs for both proliferating and quiescent cells and does not diminish
in the face of repeated doses.

170. Increasing the source-to-surface distance (SSD) for electrons will NOT affect the:

A. output.
B. penumbra.
C. treatment time.
D. percent depth dose.

THIS ITEM WAS PARTIALLY SCORED (BOTH OPTIONS A & D SCORED CORRECT).
RATIONALE: Since electrons are not deeply penetrating, there is a minimal inverse square
component. Therefore, percent depth dose (PDD) does not change with increasing source-to-
surface distance (SSD).

171. Which of the following types of colonic polyp is most likely to progress to carcinoma?

A. Hyperplastic polyp
B. Tubular adenoma
C. Tubulovillous adenoma
D. Villous adenoma

Correct answer is D. RATIONALE: Villous adenomas have a 15% to 25% chance of


progressing to carcinoma.

48 American College of Radiology


172. According to the MRC trial by Fossa, et al, what was the local recurrence rate in the
pelvis in patients with stage I testicular seminoma who received radiation therapy to the
paraaortic lymph nodes only?

A. <2%
B. 03% to 5%
C. 07% to 9%
D. 10% to 15%

THIS ITEM WAS PARTIALLY SCORED (BOTH OPTIONS A & B SCORED CORRECT).
RATIONALE: There were eighteen relapses, nine in each treatment group. Among these, four
(out of 236 patients randomized to the paraaortic radiation therapy group) were pelvic relapses,
all occurring after radiation therapy to the paraaortic lymph nodes.

173. According to RTOG 79-20 (Rotman), which of the following outcomes at 10 years is
most likely to be associated with prophylactic extended-field irradiation of the paraaortic
lymph nodes in patients with stage IIB and bulky stages IB and IIA cervical cancer?

A. There was a statistically significant difference in overall survival but no difference in


disease-free survival for the arm that received pelvic and paraaortic irradiation.
B. Survival following first failure was significantly higher in the pelvic radiation only arm.
C. A higher percentage of local failures were salvaged long-term in the pelvic radiation
only arm.
D. The cumulative incidence of death due to cervical cancer was significantly higher in
the arm that received pelvic and paraaortic irradiation.

Correct answer is A. RATIONALE: A statistically significant difference in overall survival was


found at 10 years for the pelvic and paraaortic irradiation arm, without a difference in disease-
free survival. The conclusion was that this could be explained by a lower incidence of distant
failure in complete responders and a better salvage in the complete responders who later failed
locally. Reference: Journal of the American Medical Association (JAMA). 1995 (2
Aug);274(5):387-393.

Radiation Oncology In-Training Exam 2007 49


174. Which of the following statements most appropriately justifies the minimum 6-hour
interfraction interval for hyperfractionated radiation therapy?

A. Shorter interfraction intervals maximize cell cycle redistribution effects.


B. Shorter interfraction intervals minimize the potential for tumor cell repopulation.
C. Acutely hypoxic cells reoxygenate more quickly and completely when dose fractions
are closely spaced.
D. The potential for complete repair of DNA damage is maximized in normal tissues at
risk.

Correct answer is D. RATIONALE: In radiation therapy treatments involving more than one
dose fraction per day (hyperfractionation or accelerated fractionation), it is critical to ensure that
the fractions are delivered far enough apart in time to allow all (or most, at a minimum) of the
sublethal DNA damage registered by the first dose to be repaired before the second dose is
given. Otherwise, the tolerance dose for the normal tissue(s) at risk of a radiation-induced
complication may be lower than expected. Although the half-times of repair for mammalian cells
maintained in culture tend to be short (about 0.5 to 1.0 hours, on average, although this is
somewhat variable), they are much longer in normal human tissues in vivo and may be
especially long – several hours – in late-responding normal tissues. During the 1990’s, both
EORTC Trial #22851 and the CHART clinical trials were designed to test multiple-dose-per-day
hyperfractionation and accelerated fractionation compared to conventional fractionation in
advanced head and neck cancer. Subsets of patients enrolled in these trials received their
multiple daily doses delivered at 4-hour intervals, and it soon became clear that the incidence of
spinal cord myelopathy was significantly worse than expected. When the interfraction interval
was increased to 6 hours, the incidence of late effects decreased, although arguably, it was still
higher than predicted based on the linear-quadratic model with its underlying assumption of
complete repair of sublethal damage between doses. These trials helped to establish today’s
standard of care of making sure that the time between fractions in any hyperfractionated or
accelerated fractionation regimen involving multiple doses per day is at least 6 hours.
References: Hall and Giaccia. Radiobiology for the Radiologist. 6th edition. 2006; Chapter 22,
p 388. Dische, et al. Radiotherapy Oncology. 1997;44:123; Horiot, et al. Radiotherapy
Oncology. 1997;44:111.)

175. What is the best photon energy for total-body irradiation?

A. 01.25 MV
B. 06 MV
C. 15 MV
D. 18 MV

Correct answer is B. RATIONALE: Cobalt would require high dmax doses, 15 MV would
introduce neutrons, and 18 MV would require bolus or thick spoilers.

50 American College of Radiology


176. Which of the following statements about the EORTC hypopharynx preservation trial is
true?

A. Fifty percent of the patients had primary tumors involving the pyriform sinus.
B. The overall survival at 5 years revealed a significant improvement associated with
chemotherapy, followed by radiation therapy.
C. Only patients with a complete clinical response after induction chemotherapy were
eligible for definitive radiation therapy.
D. Patients with palpable adenopathy were required to undergo neck dissections
following completion of radiation therapy.

Correct answer is C. RATIONALE: This is one of the fundamental differences between this trial
and the VA laryngeal preservation study. Patients were required to have achieved a clinical
complete response to induction chemotherapy prior to undergoing radiation therapy.

177. Which of the following statements about TNM staging for lung cancer is FALSE?

A. Stage N1 involves lymph node stations 10-14.


B. Stage N2 involves the ipsilateral mediastinal lymph nodes.
C. Stage N3 involves the contralateral hilar and mediastinal lymph nodes.
D. Stage M1 involves the contralateral supraclavicular lymph nodes.

Correct answer is D. Reference: AJCC Cancer Staging Manual.

178. Within a target volume, the following 11 absorbed doses in water points (in centigray)
represent the prescribed absorbed dose in water:

4500, 4520, 4520, 4540, 4540, 4550, 4540, 4540, 4530, 4510, and 4500.

The difference between the mean target volume and the modal target volume of the
absorbed dose in water is approximately how many centigray (cGy)?

A. 04 cGy
B. 10 cGy
C. 14 cGy
D. 20 cGy

Correct answer is C. RATIONALE: Requires the resident to distinguish mean target volume
from modal target volume of absorbed dose in water.

Radiation Oncology In-Training Exam 2007 51


179. What is the most appropriate therapy for a 6-year-old child with a completely resected
medulloblastoma with no evidence of metastasis?

A. Craniospinal irradiation to a dose of 23.4 Gy and 55.8-Gy boost dose to the


posterior fossa
B. Concurrent chemotherapy with craniospinal irradiation to 23.4 Gy and 55.8-Gy boost
dose to the posterior fossa, followed by additional chemotherapy
C. Concurrent chemotherapy with craniospinal irradiation to 36 Gy and 55.8-Gy boost
dose to the posterior fossa, followed by additional chemotherapy
D. Induction chemotherapy, craniospinal irradiation to a dose of 36 Gy and 55.8-Gy
boost dose to the posterior fossa, followed by additional chemotherapy

Correct answer is B. RATIONALE: Standard of care in North America currently is concurrent


Vincristine with craniospinal irradiation (CSI) to a dose of 23.4 Gy and a boost to the posterior
fossa followed by additional chemotherapy for standard risk patients. This question is asking
the examinee about risk groups (standard versus high risk) and the appropriate treatment of
standard-risk patients.

180. Which of the following statements about asymmetrical versus symmetrical chromosomal
aberrations is FALSE?

A. Asymmetrical chromosomal aberrations are more likely to be carcinogenic than


fatal.
B. Symmetrical chromosomal aberrations tend to be longer-lived than asymmetrical
ones.
C. A dicentric chromosome is a type of asymmetrical chromosomal aberration.
D. A chromosome inversion is a type of symmetrical chromosomal aberration.

Correct answer is A. RATIONALE: An asymmetrical chromosomal aberration results in the


formation of an acentric fragment that is often lost from the cell at the subsequent mitosis,
usually leading to cell death. Symmetrical aberrations result in an even exchange of
chromosomal material with nothing lost, and as such, tend to be less fatal, longer-lived, and
potentially more carcinogenic.

181. Which of the following regions of the body would require a boost field for total skin
electron therapy using a 6-field, modified Stanford technique?

A. Scalp
B. Soles of the feet
C. Lens of the eye
D. Fingernails

Correct answer is B. RATIONALE: The scalp often has higher doses due to the tangential
effect, while the lens and fingernails are often protected.

52 American College of Radiology


182. Which of the following statements about squamous cell carcinoma of the larynx is true?

A. Primary tumors more commonly arise from the supraglottic laryngeal structures than
from the glottic region.
B. At the time of diagnosis, 50% of tumors have extended beyond the laryngeal
structures.
C. Supraglottic tumors are more likely to present with palpable lymphadenopathy than
glottic lesions.
D. Subglottic tumors tend to present with early lesions that are easily managed with
primary surgical resection.

Correct answer is C. RATIONALE: Glottic lesions have a lower chance than supraglottic
lesions of spreading to the lymph nodes, because the true vocal cords have essentially no
capillary lymphatics. In fact, there is a 20% chance that patients with stage T3 and T4 glottic
cancers will have clinically involved lymph nodes at initial presentation; whereas, there is a 55%
chance that patients with stage T1-T4 supraglottic cancer will have clinically positive lymph
nodes at diagnosis.

183. Which of the following statements about breast cancer in men is true?

A. The tumor is most commonly ER-positive.


B. The disease should be treated with cisplatin-based chemotherapy.
C. The majority of patients who undergo genetic screening will be found to have a
BRCA1 germline mutation.
D. Approximately 25% of patients will have noninvasive disease at diagnosis.

Correct answer is A. RATIONALE: Approximately 80% to 90% of breast cancers in men are
ER-positive. Management of breast cancer in men parallels that of breast cancer in women.
The BRCA2 germline mutation is most frequently found in men who have breast cancer.
Approximately 5% of men with breast cancer have noninvasive disease at diagnosis.
Reference: Perez and Brady. Locally Advanced Breast Cancer, Chapter 50 (Marie Taylor, MD,
et al), 2004; pp 1541-1543.

184. What is the minimum percentage of oligodendroglioma cells required for a tumor to be
classified as a mixed oligoastrocytoma?

A. 05%
B. 15%
C. 25%
D. 50%

Correct answer is C. RATIONALE: For a tumor to be classified as a mixed oligoastrocytoma,


at least 25% of the tumor has to consist of oligodendroglioma cells.

Radiation Oncology In-Training Exam 2007 53


185. Why did the A-bomb survivors from Hiroshima show a higher yield per gray of stable,
exchange-type chromosomal aberrations in lymphocytes than the A-bomb survivors
from Nagasaki?

A. The median age of the Japanese survivors in Hiroshima was approximately half that
of those in Nagasaki.
B. A disproportionately higher number of Japanese survivors were located closer to the
epicenter of the explosion in Hiroshima than in Nagasaki.
C. The atomic bomb dropped on Hiroshima produced more neutrons than the bomb
dropped on Nagasaki.
D. The source of drinking water contained higher concentrations of naturally occurring
antioxidants in Nagasaki than in Hiroshima.

Correct answer is C. RATIONALE: The radiation dose response curve for the induction of
stable, exchange-type chromosomal aberrations was both steeper and more linear for
Hiroshima versus Nagasaki survivors because the Hiroshima bomb produced more high-LET
radiation (neutrons).

186. How is optimal field definition accomplished for electron arc therapy?

A. Custom cerrobend shielding in an applicator


B. Custom lead shielding on the skin
C. Conventional multileaf collimators
D. Collimating jaws

Correct answer is B. RATIONALE: For electron treatments, the closer the field shaping is to
the patient, the sharper the penumbra. In addition, skin shielding moves with the patient.

187. Which of the following statements about patients who have HIV and anal cancer is
FALSE?

A. HAART has been shown to reduce the severity and incidence of cancer-related
toxicity.
B. The risk of increased toxicity with chemotherapy and radiation therapy is greater in
patients with CD4 cell counts that are less than 200.
C. Patients with HIV are at increased risk for toxicity associated with chemotherapy and
radiation therapy.
D. Patients with HIV are at increased risk for the development of anal intraepithelial
neoplasia and squamous cell carcinoma of the anal canal.

Correct answer is A. RATIONALE: HAART has not been shown to reduce the severity and
incidence of cancer-related toxicity.

188. Which stage is most appropriate for a patient who has testicular seminoma involving the
spermatic cord with multiple 2- to 5-cm paraaortic lymph nodes?

A. Stage IB
B. Stage IIB
C. Stage IIC
D. Stage IIIA

Correct answer is B. Reference: AJCC Cancer Staging Manual, 6th edition.

54 American College of Radiology


189. According to GOG 92 (Sedlis), which of the following statements about the addition of
postoperative adjuvant pelvic radiation therapy for patients with stage IB cervical
carcinoma with poor prognostic features is true?

A. It did not benefit patients with adenocarcinoma.


B. It did not improve the progression-free survival rate.
C. It was beneficial for patients with invasion of the deep third of the cervical stroma
and lymphovascular space invasion.
D. It significantly reduced the risk of recurrence by 80%.

Correct answer is C. References: Rotman, et al. International Journal of Radiation Oncology,


Biology, Physics. 2006;65(1):169-176. Sedlis, et al. Gynecologic Oncology. 1999;73:177-183.

190. Which of the following subsites of the hypopharynx is correctly ordered from the least to
the most commonly involved sites?

A. Pyriform sinus, posterior pharyngeal wall, postcricoid region


B. Posterior pharyngeal wall, postcricoid region, pyriform sinus
C. Posterior pharyngeal wall, pyriform sinus, postcricoid region
D. Postcricoid region, posterior pharyngeal wall, pyriform sinus

Correct answer is D. RATIONALE: Postcricoid region tumors are not very common. Pyriform
sinus cancers are more common than posterior pharyngeal tumors.

191. Which of the following statements about malignant mesothelioma is true?

A. Chemoradiation therapy alone offers promising results.


B. Most patients die of metastatic disease and not of local recurrence.
C. Patients who have the epithelial subtype have a worse prognosis.
D. Mediastinal lymph node involvement affects the prognosis.

Correct answer is D. Reference: Journal of Clinical Oncology (JCO). 1993;11:1172-1178.

192. Hodgkin lymphoma most commonly occurs during which decade of life?

A. Second
B. Third
C. Fourth
D. Fifth

Correct answer is B. RATIONALE: The majority of Hodgkin lymphoma cases occur during the
third decade (20 to 30 years of age). The incidence is bimodal, though, with a second peak in
incidence (although much smaller) after 50 years of age or in the sixth decade onward.

Radiation Oncology In-Training Exam 2007 55


193. Which of the following tumors is LEAST likely to be treated with craniospinal irradiation?

A. Pineoblastoma
B. Ependymoblastoma
C. Nongerminomatous germ cell tumor
D. Pure germinoma

Correct answer is D. RATIONALE: Pure germinomas have the lowest risk for dissemination to
the spine and can be treated with whole ventricular irradiation only. The other tumors have a
much higher risk of spread through the cerebrospinal fluid (CSF), requiring craniospinal axis
irradiation.

194. Which of the following statements about the RBE for neutrons is true?

A. For neutrons of intermediate LET, the oxygen enhancement ratio (OER) equals 1.0.
B. Neutron irradiation reduces the natural variability in radiation sensitivity among
different mammalian cell types.
C. The RBE is higher for a severe endpoint (low level of cell survival) than for a milder
endpoint (high level of cell survival).
D. For a given endpoint, the RBE is higher for single radiation doses than for
fractionated doses.

Correct answer is B. RATIONALE: Different mammalian cell types, apparently normal in their
capacity to detect and repair DNA damage, exhibit a relatively broad range of radiosensitivities
to low-LET radiations (such as X-rays) when assayed in vitro, with SF2 values ranging from as
high as 0.8 to as low as 0.15. Much, although not all, of this difference can be accounted for by
differences in the width of the shoulder region of the radiation survival curve. As the LET of the
type of radiation increases, survival curves become steeper and the width of the shoulder region
becomes narrower, until the shoulder region is eliminated completely for radiations
characterized by LET radiations in the range of 100 keV/µm, and the survival curve as a whole
becomes an exponential function of dose. Because of this, the natural variability in radiation
sensitivity between different mammalian cell types characteristic of their X-ray dose response is
“dampened” (although not completely eliminated) for higher-LET radiations, such as neutrons.
References: Hall and Giaccia. Radiobiology for the Radiologist, 6th edition, 2006; Chapter 7,
pp 110-111. Steel. Basic Clinical Radiobiology, 3rd edition, 2002; Chapter 19, pp 209-211.

195. Prescriptions for radiosurgery are made to what percent of maximum dose for the listed
modalities?

Gamma Knife Linac Beam


A. 100% 100%
B. 080% 050%
C. 050% 080%
D. 050% 050%

Correct answer is C. RATIONALE: The cobalt Gamma Knife sources provide a wider
penumbra than the sharply collimated linac beams.

56 American College of Radiology


196. Soft tissue sarcoma most commonly arises from what site?

A. Head and neck


B. Retroperitoneum
C. Lower extremity
D. Upper extremity

Correct answer is C. RATIONALE: By far, the most common location is the lower extremity,
which is more prevalent than the upper extremity. Head and neck rhabdomyosarcoma accounts
for ≤10% of cases in most series.

197. Which of the following adjuvant systemic therapies would be most appropriate to
recommend for an otherwise healthy patient who has HER-2/neu-positive (FISH)
ER/PR-negative breast cancer with positive axillary lymph nodes?

A. Four cycles of dose-dense doxorubicin and cyclophosphamide (AC) in conjunction


with four cycles of paclitaxel.
B. Four cycles of AC, four cycles of paclitaxel with concurrent trastuzumab, and
continuation of trastuzumab for 1 year.
C. Four cycles of paclitaxel, four cycles of AC with concurrent trastuzumab, and
continuation of trastuzumab for 1 year.
D. Six cycles of docetaxel, doxorubicin, and cyclophosphamide.

Correct answer is B. RATIONALE: Trastuzumab has been shown to improve disease-free


survival/overall survival for women with HER-2/neu-positive breast cancer and is now
recommended for use as adjuvant therapy. Trastuzumab is generally combined with
chemotherapy, but it is not scheduled to be administered with doxorubicin due to the potential
for additive cardiac toxicity.

198. Which of the following cell types would produce the highest neutron RBE, assuming
otherwise identical irradiation and physiological conditions?

A. Melanoma
B. Bone marrow
C. Mammary epithelium
D. Germinal cells

Correct answer is A. RATIONALE: In general, relative biological effectiveness (RBE) values


are high for tissues that accumulate and repair a great deal of sublethal damage and low for
those that do not. Reference. Hall and Giaccia. Radiobiology for the Radiologist, 6th edition,
2006; Chapter 7, p 113.

199. What is the purpose of a circulator in a linear accelerator?

A. Absorb reflected microwaves


B. Adjust the frequency of microwaves
C. Adjust the frequency of electron injection
D. Regulate the flow rate of cooling water

Correct answer is A. RATIONALE: This item requires knowledge of the purpose of a circulator
in a linear accelerator.

Radiation Oncology In-Training Exam 2007 57


200. Which of the following MRI sequences and margins would be most appropriate for target
delineation of a low-grade oligodendroglioma?

A. T1 + 1.5 cm
B. T1 + 3 cm
C. T2 + 2 cm
D. FLAIR + 3 cm

Correct answer is C. RATIONALE: The MRI sequence and margin T2 + 2 cm are the most
appropriate for delineation of a low-grade oligodendroglioma that has been used in the RTOG
trials.

201. Which of the following tumor stages is most likely to be associated with a patient who
has a 3-cm squamous cell cancer of the anus with direct rectal wall invasion?

A. T1
B. T2
C. T3
D. T4

Correct answer is B. Reference. AJCC Cancer Staging Manual, 6th edition.

202. Which histology is most likely in men older than 60 years with testicular cancer?

A. Yolk sac tumor


B. Classical seminoma
C. Choriocarcinoma
D. Lymphoma

Correct answer is D. RATIONALE: Lymphoma is the most common histology for a testicular
mass in men older than 60 years of age.

58 American College of Radiology


203. Which of the following outcomes is most likely to be associated with the GOG 85/SWOG
8695 Intergroup protocol study (Whitney) comparing primary radiation therapy with
hydroxyurea versus radiation therapy with fluorouracil and cisplatin for patients with
locally advanced cervical carcinoma?

A. Severe or life-threatening leukopenia was more common in the group receiving


fluorouracil with cisplatin.
B. Patients in the fluorouracil with cisplatin treatment arm had better overall and
progression-free survival rates.
C. Patients with positive paraaortic lymph nodes at the time of lymphadenectomy
achieved the greatest benefit in overall survival rates.
D. Patients in the hydroxyurea treatment arm experienced fewer pelvic recurrences.

Correct answer is B. RATIONALE: The combination of 5-FU and cisplatin with radiation
therapy offers patients better progression-free survival and overall survival than treatment with
hydroxyurea (HU). Patients underwent staging paraaortic lymphadenectomies. Patients with
metastases were excluded from participation in the trial. Note that a pelvic lymphadenectomy
was not required. Patients in the CF arm had fewer pelvic recurrences. Reference: Whitney, et
al: Journal of Clinical Oncology (JCO). 1999 (May);17(5):1339-48.

204. Which of the following factors is NOT typically associated with improved overall survival
for patients who present with unknown primary tumors of the head and neck region?

A. Radiation doses of >50 Gy


B. Complete resection of the involved lymph nodes
C. Subsequent presentation of the primary tumor
D. Stage N1 versus stage N2 lymph node involvement

Correct answer is C. RATIONALE: The subsequent development of a primary tumor in a


patient who has undergone postoperative radiation therapy portends a worse prognosis.

205. Which of the following conditions should be treated like small cell lung cancer?

A. Combined small cell lung cancer and non-small cell lung cancer
B. Non-small cell lung cancer with neuroendocrine features
C. Large cell neuroendocrine tumors
D. Atypical carcinoid tumors

Correct answer is A. Reference. Journal of the National Comprehensive Cancer Network


(JNCCN). 2006;4:614.

206. Which of the following eye tumors should be diagnosed clinically without a biopsy?

A. Retinoblastoma
B. Orbital rhabdomyosarcoma
C. Conjunctival lymphoma
D. Pseudolymphoma

Correct answer is A. RATIONALE: A biopsy of retinoblastoma can result in vitreous seeding.


207. Which of the following statements about CNS germinomas is true?

A. The suprasellar region is the most common site of origin.


B. They most commonly occur during the second and third decade of life.

Radiation Oncology In-Training Exam 2007 59


C. The best treatment is craniospinal axis irradiation, followed by a boost dose to the
primary tumor.
D. Alpha-fetoprotein serum levels are elevated in patients with pure germinoma.

Correct answer is B. RATIONALE: Patients with CNS germinomas most commonly present in
their 2nd decade of life. The pineal region is the most common site of origin. Because the
predominant risk for its dissemination is in the ventricles, patients can be treated with whole
ventricle radiation therapy, followed by primary tumor boost avoiding full craniospinal axis
irradiation. Beta-HCG can sometimes be slightly elevated in patients with pure germinoma, but
elevated alpha-fetoprotein serum levels will exclude that diagnosis.

208. For many biological endpoints, the linear energy transfer (LET) at which the relative
biological effectiveness (RBE) reaches its highest value is:

A. 0001 keV/µm.
B. 0010 keV/µm.
C. 0100 keV/µm.
D. 1000 keV/µm.

Correct answer is C. RATIONALE: At a LET of approximately 100 keV/µm, the average


density of ionization events along an incident particle's track roughly corresponds to the
diameter of the DNA double helix, meaning a higher probability of a double-strand break being
produced than at either lower or higher LET.

209. When used to calibrate a linear accelerator, the Pion factor refers to ion collection
efficiency as a function of ion chamber:

A. voltage magnitude.
B. voltage polarity.
C. wall material.
D. radius.

Correct answer is A. RATIONALE: Requires knowledge of calibration protocol.

210. Which of the following tests is used to determine the difference in survival between two
treatment regimens?

A. Chi-square test
B. Fisher exact test
C. Log-rank test
D. Student t test

Correct answer is C. RATIONALE: The log-rank test is the most common method of
comparing independent groups of survival times.

60 American College of Radiology


211. The basal molecular phenotype of breast cancer most commonly is:

A. ER-positive, PR-positive, HER-2/neu-negative.


B. ER-positive, PR-positive, HER-2/neu-positive.
C. ER-negative, PR-negative, HER-2/neu-negative.
D. ER-negative, PR-negative, HER-2/neu-positive.

Correct answer is C. RATIONALE: DNA microarray analysis has enabled tumor gene
expression subtypes for breast cancer that fall into the following categories: Gene profile by
hormone receptors and HER-2/neu status enable recommendations regarding treatment
programs.
Luminal A → ER+/PR+, HER-2/neu-negative
Luminal B → ER+/PR+, HER-2/neu-positive
Basal-like → ER/PR and HER-2/neu-negative
HER-2/neu-positive → ER/PR-negative, HER-2/neu-positive
Reference: Race, breast cancer Subtypes & survival in the Carolina breast cancer study.
Journal of the American Medical Association (JAMA). 2006;295:2492-2502.

212. A 22-year-old man with Hodgkin disease has adenopathy of the right cervical and right
supraclavicular regions and weight loss of >10% of his baseline body weight. Which of
the following cancer stages is most likely to be associated with this patient's condition?

A. Stage IA
B. Stage IB
C. Stage IIA
D. Stage IIB

Correct answer is B. RATIONALE: The patient has significant weight loss and night sweats,
both of which qualify as "B" symptoms. The cervical and supraclavicular lymph node chains are
considered a unified area for the purpose of staging and management. Hence, the patient has
cancer stage IB.

213. Which of the following statements about pancreatic cancer is FALSE?

A. Approximately 10% to 20% of pancreatic cancers are associated with hereditary


factors.
B. New-onset diabetes mellitus may be the first clinical feature in 10% of patients.
C. Tumors of the pancreatic head arise to the right of the superior mesenteric
vein-portal vein confluence and include tumors of uncinate origin.
D. The most common physical finding at initial presentation is Courvoisier's sign.

Correct answer is D. RATIONALE: The most common physical finding at initial presentation is
jaundice.

Radiation Oncology In-Training Exam 2007 61


214. Which of the following statements about proton radiation therapy is FALSE?

A. A favorable characteristic is the virtual absence of an exit dose.


B. Its physical dose distributions make it effective for treating tumors close to critical
normal tissues.
C. It is likely to be beneficial for treating slow-growing tumors with sizeable hypoxic
fractions.
D. Neutrons are harder to shield than protons.

Correct answer is C. RATIONALE: The relative biological effectiveness (RBE) of protons


possesses radiobiological properties that are not significantly different from those of traditional
X-rays or gamma rays. As such, the proton oxygen enhancement ratio (OER) would still be
high, and hypoxic tumors would not differentially benefit from proton radiation therapy.

215. The efficiency of x-ray production via the bremsstrahlung process is proportional to the
product of:

A. Z and V.
B. Z2 and V.
C. Z and V2.
D. Z2 and V2.

THIS ITEM WAS NOT SCORED. RATIONALE: Requires knowledge of the bremsstrahlung
process.

216. What percent of patients with bladder cancer will require a cystectomy for
radiation-related complications after bladder preservation therapy with concurrent
chemoradiation?

A. <5%
B. 5% to 10%
C. 11% to 15%
D. >15%

Correct answer is A. RATIONALE: Fewer than 1% of patients undergoing bladder-conserving


therapy will require cystectomy for persistent treatment-related symptoms.

62 American College of Radiology


217. Which of the following findings is most likely to be associated with GOG 120 (Rose) that
studied the use of radiation therapy in combination with three concurrent chemotherapy
regimens for patients with locally advanced cervical cancer?

A. Stage IB1 cancer treated with cisplatin, fluorouracil, and hydroxyurea was
associated with a better local control rate.
B. The three-drug regimen had the lowest combined frequency of grade 3 and grade 4
side effects.
C. Cisplatin-based chemotherapy improved the survival rate and the progression-free
survival rate.
D. The overall survival rate was highest in the hydroxyurea only treatment arm.

Correct answer is C. RATIONALE: The study evaluated stages IIB-IVA cervical cancer
patients without paraaortic lymph node involvement. It was a randomized trial of cisplatin alone,
5-FU + cisplatin + hydroxyurea, and hydroxyurea alone. All patients received radiation therapy.
Regimens of radiation therapy and chemotherapy that contain cisplatin improve the rates of
survival and progression-free survival. The three drug regimens had the highest frequency of
combined grade 3 and 4 side effects. Reference: Rose, et al. Concurrent cisplatin-based
radiotherapy and chemotherapy for locally advanced cervical cancer. New England Journal of
Medicine (NEJM). 1999 ( April);340(15):1144-1153.

218. Which of the following statements about unknown primary head and neck tumors is
true?

A. PET imaging is able to detect the primary tumor in 60% of cases.


B. Random biopsies will reveal the primary lesion in 40% of cases.
C. Chest imaging will reveal a primary lung tumor in 20% of cases.
D. Tonsillectomy may reveal the occult primary tumor in 20% of cases.

Correct answer is D. RATIONALE: A tonsillectomy can reveal occult primary tumors in 20% of
cases. A PET scan is able to detect the primary tumor in approximately 15% to 20% cases
only.

219. Taxane derivatives elicit cytotoxic effects mainly by which of the following mechanisms?

A. Stabilization of microtubules and prevention of their disassembly


B. Intercalation into DNA and inhibition of DNA replication
C. Inhibition of topoisomerases
D. Inhibition of DNA repair

Correct answer is A. RATIONALE: Taxanes, such as paclitaxel and docetaxel, shift the
dynamic equilibrium between tubulin dimers and microtubule assembly, and they actually
stabilize the tubulin molecules in microtubles, resulting in the stabilization of microtubules
against depolymerization.

Radiation Oncology In-Training Exam 2007 63


220. A permanent implant is removed after 30 days. The radionuclide used for the implant
has a half-life of 6 days. What percent of the original planned dose still needs to be
delivered?

A. 2%
B. 3%
C. 4%
D. 6%

Correct answer is B. RATIONALE: Requires knowledge of the use of half-life for calculations.

221. Which of the following radiation doses administered concurrently with chemotherapy is
considered to be the most INAPPROPRIATE treatment for limited-stage small cell lung
cancer?

A. 45 Gy at 1.5 Gy per fraction twice daily


B. 45 Gy at 1.8 Gy per fraction
C. 50 Gy at 2.0 Gy per fraction
D. 60 Gy at 2.0 Gy per fraction

Correct answer is B. Reference. Journal of the National Comprehensive Cancer Network


(JNCCN). 2006;4:613.

222. Which of the following radiation doses were compared in the randomized treatment arms
of the German Hodgkin Study Group, HD 10 trial of patients with early-stage disease?

A. 36 Gy vs. 40 Gy
B. 24 Gy vs. 36 Gy
C. 20 Gy vs. 30 Gy
D. 00 Gy vs. 20 Gy

Correct answer is C. RATIONALE: The randomization of radiation therapy is 20 Gy vs. 30 Gy,


which followed a randomization of 2 versus 4 cycles of ABVD systemic therapy.

223. Which of the following statements about brainstem gliomas is true?

A. High-grade tumors display exophytic growth.


B. The majority of tumors are low-grade gliomas.
C. Radiation doses greater than 54 Gy have not been shown to improve survival.
D. The addition of chemotherapy has improved overall survival rates.

Correct answer is C. RATIONALE: The majority of brainstem tumors are high-grade although
the exact grade distribution is difficult due to the lack of biopsy information. High-grade tumors
mostly demonstrate intrinsic growth into the brainstem. The POG randomized trial of
conventional vs. hyperfractionated high-dose irradiation failed to show any benefit from radiation
dose escalation. There has been no benefit shown from the addition of chemotherapy for
brainstem gliomas.

64 American College of Radiology


224. Sulfhydryl-containing compounds, such as amifostine, are effective dose modifiers
because of their ability to:

A. scavenge free radicals.


B. bind to caspases and directly inhibit their activation.
C. form free-radical intermediates that interact with DNA.
D. sensitize hypoxic cells regardless of the radiation dose.

Correct answer is A. RATIONALE: Amifostine is believed to protect cells from ionizing


radiation by scavenging free radicals and by facilitating restitution of target radicals through
donation of hydrogen.

225. Which of the following sealed radioactive sources has the most anisotropic dose
distribution?
125
A. I
137
B. Cs
192
C. Ir
198
D. Au

Correct answer is A. RATIONALE: 125I has a much lower energy, and thus, the source
radiation is absorbed in the source capsule, leading to anisotropy (less dose along the source
axis).

226. Which of the following statements about patients who are diagnosed with breast cancer
and have supraclavicular lymph node involvement is true?

A. The diagnosis should be classified as stage M1 breast cancer.


B. Approximately 30% of patients will have clinically detectable internal mammary
lymph nodes on CT scan of the chest.
C. Over 20% of patients will have lymphedema and brachial plexopathy.
D. Over 20% of patients will survive 10 years with aggressive trimodality therapy.

Correct answer is D. RATIONALE: A study by Brito, et al, provides evidence that aggressive
treatment of patients with positive supraclavicular lymph nodes (SCLN) results in outcomes
comparable to those in patients with locally advanced breast cancer (LABC), stage IIIB, without
distant metastases. In this study, 70 patients with SCLN-positive LABC received intensive
treatment that included induction chemotherapy, surgery, post-surgical chemotherapy, and
irradiation. At a median follow-up time of 8.5 years, there was no difference in disease-free
survival or overall survival in LABC patients with positive SCLN and no other sign of distant
metastasis compared with stage IIIB patients without distant metastasis. Both stage IIIB and
SCLN-positive patients differed significantly in overall survival when compared with stage IV
patients. These findings indicate that classifying SCLN as a distant metastasis may be a
disservice to patients, because it implies incurability and may lead to suboptimal therapy.
Patients with ipsilateral SCLN metastases and no other distant metastases should be classified
as having stage N3 rather than stage M1 disease, because their clinical course and outcomes
are similar to patients with stage IIIB LABC. To clarify the significance of stage N3 disease, a
new category of stage IIIC has been instituted. References: Brito, RA, et al. Long-term results
of combined modality therapy for locally advanced breast cancer with ipsilateral supraclavicular
metastases. Journal of Clinical Oncology (JCO). University of Texas, MD Anderson Center
Experience, 2001;19:628-639. AJCC Cancer Staging Manual. (6th edition), 2002; p 233.

Radiation Oncology In-Training Exam 2007 65


227. Which of the following procedures will have the greatest effect in decreasing CT image
noise?

A. Increasing the kilovolt peak by 20%


B. Increasing the milliamperes by 20%
C. Decreasing the slice thickness to 1 mm from 3 mm
D. Decreasing the field of view to 30 cm from 48 cm

Correct answer is B. RATIONALE: Increasing kVp increases x-ray production efficiency


slightly as well as detected photons due to decreased attenuation, but increasing current (mA)
more significantly improves photon statistics. Decreasing slice thickness increases noise (fewer
detected photons per detector element). Decreasing field of view has minimal effect on noise.

228. Which of the following statements about uveal melanomas is true?

A. Systemic metastases are the cause of death in the majority of patients.


B. A biopsy is necessary to establish the diagnosis in the majority of patients.
C. Preoperative radiation therapy does not improve local control in patients with large
tumors.
D. Enucleation produces better overall survival than plaque brachytherapy in patients
with medium-sized tumors.

Correct answer is C. RATIONALE: COMS showed that both enucleation and plaque
brachytherapy produces the same overall survival outcome. Preoperative radiation therapy
before enucleation does not produce better local control compared to enucleation alone.

229. The oxygen enhancement ratio (OER) for high-LET radiations such as alpha particles is
approximately:

A. 1.0.
B. 1.5.
C. 2.0.
D. 2.5.

Correct answer is A. RATIONALE: The oxygen enhancement ratio (OER) for high-LET
radiations, such as alpha particles, is essentially unity, since there is no oxygen effect noted
with such densely ionizing radiations. This is because most of the damage from high-LET
radiation is mediated by direct effects rather than indirect effects.

230. Which of the following factors will cause a dose delivery error of >5% for a planned
single-field treatment with 6-MV photons at a source-to-skin distance of 100 cm and
prescription depth of 5 cm?

A. Change in depth of 2 cm
B. Change in source-to-skin distance of 2 cm
C. Maximum tolerable variation in daily output
D. Offset of 0.5 cm along the direction of a 15° wedge

Correct answer is A. RATIONALE: The use of 6-MV photons change the radiation dose by
approximately 6% per centimeter past dmax due to attenuation and inverse square falloff.

66 American College of Radiology


231. In what region are cholangiocarcinomas most commonly located?

A. Distal
B. Proximal
C. Intrahepatic
D. Hilar

Correct answer is D. RATIONALE: Sixty-seven percent of cholangiocarcinomas occur in the


hilar region.

232. Bladder cancer associated with hydronephrosis and invasion limited to the outer half of
the bladder wall corresponds to which T stage?

A. T2
B. T3
C. T4a
D. T4b

Correct answer is A. RATIONALE: Hydronephrosis does not affect clinical T staging in bladder
cancer.

233. Based on the GOG prospective surgical-pathological study (Delgado), what is the 3-year
disease-free interval following a radical hysterectomy for a patient with pathologic stage
IB squamous cell carcinoma of the cervix with invasion of the deep third of the cervical
stroma?

A. 94%
B. 85%
C. 74%
D. 53%

Correct answer is C. RATIONALE: Delgado, et al, reported the findings of a GOG prospective
surgical-pathological study of disease-free interval (DFI) in patients with stage IB squamous cell
carcinoma of the cervix. The DFI was strongly correlated with depth of stromal invasion. In
fractional terms, the DFI was 94.1% for superficial third, 84.5% for middle third, and 73.6% for
deep third invasion. This study laid the ground work for postoperative adjuvant radiation
therapy for stage IB cancers of the cervix. Reference: Gynecologic Oncology, 1990
(September);38(3):352-357.

234. Adverse predictors of overall survival for patients with parotid gland malignancies
include all of the following factors EXCEPT:

A. deep lobe lesion.


B. acinic cell histology.
C. high-grade mucoepidermoid histology.
D. facial nerve paralysis.

Correct answer is B. RATIONALE: Acinic cell histology is not an adverse factor because it is a
low-grade tumor.

Radiation Oncology In-Training Exam 2007 67


235. What is the most sensitive gestational period for prenatal death after irradiation?

A. Preimplantation
B. Major organogenesis
C. Minor organogenesis
D. Fetal

Correct answer is A. RATIONALE: Neonatal death, rather than prenatal death, may occur
during the fetal period or major organogenesis period. Higher doses are also required
compared to the preimplantation period.

236. Which of the following treatment strategies would be best for uniformly treating a target
that extends from the skin surface to 1 cm deep while sparing deeper tissue?

A. 6-MeV electrons with 0.5-cm bolus


B. 6-MeV electrons with 2-cm bolus
C. 9-MeV photons with 1-cm bolus
D. 9-MeV photons with 3-cm bolus

Correct answer is A. RATIONALE: Ideally, treatment should be with 90% or greater maximum
dose, found at depth t/4. Bolus serves to increase the surface dose, as well as position the
edge of steep falloff close to the posterior target edge.

237. Which of the following treatments would NOT be appropriate after a lobectomy with
negative margins is performed for a patient who has limited-stage small cell lung
cancer?

A. Radiation therapy with positive mediastinal nodal dissection


B. Chemoradiation therapy with negative mediastinal nodal dissection
C. Chemotherapy with negative mediastinal nodal dissection
D. Chemotherapy with negative mediastinal nodal sampling

Correct answer is B. RATIONALE: There is no evidence that radiotherapy benefits the patient
with a negative mediastinal nodal dissection. Reference: Journal of the National
Comprehensive Cancer Network (JNCCN). 2006;4:610.

238. Which of the following Hodgkin lymphoma subtypes is most closely associated with the
development of non-Hodgkin lymphoma?

A. Nodular sclerosis
B. Lymphocyte depleted
C. Lymphocyte predominant
D. Lymphocyte rich

Correct answer is C. RATIONALE: Compared to other subtypes, there is a documented


association between lymphocyte-predominant Hodgkin lymphoma and non-Hodgkin lymphoma.
The risk is thought to be approximately 3% (relative risk of 1.8), and the risk is presumed to be
independent of therapy.

68 American College of Radiology


239. Which of the following primary sites is most commonly involved in patients who are
diagnosed with Ewing sarcoma?

A. Pelvis
B. Humerus
C. Fibula
D. Tibia

Correct answer is A. RATIONALE: Based on the CCG 7881/POG 8850 trial, the primary site of
involvement was the pelvis in 24% of patients with Ewing sarcoma. Involvement of the humerus
occurred in 7% of patients, the tibia in 10% of patients, and the fibula in a lower percentage of
patients than those with involvement of the humerus.

240. During gene therapy, genes are most commonly transferred into cells by:

A. viral vectors.
B. bacterial plasmids.
C. synthetic oligonucleotide primers.
D. small-interfering RNA (siRNA).

Correct answer is A. RATIONALE: Retroviruses, adenoviruses, and herpes viruses are


commonly used as vectors in gene therapy.

241. The activity of a 20-mCi 137Cs source is approximately how many milligram radium
equivalents?

A. 06 mg Ra eq
B. 08 mg Ra eq
C. 11 mg Ra eq
D. 14 mg Ra eq

Correct answer is B. RATIONALE: Requires knowledge of cesium-137 sources and mg Ra eq.

242. What would be the best reason for early termination of a randomized phase III trial?

A. A protocol violation
B. An adverse event occurring in one of the treatment arms
C. Evidence of one treatment arm showing significant superiority during the trial
D. Loss of follow-up of some of the subjects during the trial

Correct answer is C. RATIONALE: It is important to know that when one treatment arm shows
definite superiority during the trial, it is unethical to continue randomization.

243. Which of the following statements about desmoid tumors is true?

A. They commonly metastasize to the lungs.


B. They commonly metastasize to the bones.
C. Surgical excision is the most common treatment of choice.
D. Definitive radiation therapy can achieve local control in 25% of cases.

Correct answer is C. RATIONALE: Retrospective studies show good local control rates
(approx. 75% or higher) for definitive radiation therapy. These tumors rarely metastasize.
Surgery is the initial treatment of choice.

Radiation Oncology In-Training Exam 2007 69


244. The hypoxic fraction in experimental animal tumors is approximately:

A. 15%.
B. 30%.
C. 50%.
D. 70%.

Correct answer is A. RATIONALE: Although the hypoxic fraction may range from 0% to 50%,
the average obtained from surveys conducted on numerous animal tumor types is
approximately 15%.

245. A measurement is performed using an air ionization chamber. An increase in 3°C has
what effect on percent change in the reading?

A. −1%
B. −0.5%
C. +0.5%
D. +1%

Correct answer is A. RATIONALE: Requires knowledge of how temperature affects mass of air
in ionization chambers.

246. What structure in the eye is most radioresistant to late injury?

A. Lens
B. Retina
C. Lacrimal gland
D. Sclera

Correct answer is D. RATIONALE: The sclera is most radioresistant, and the lens of the eye is
the most radiosensitive of the structures described.

247. Which of the following conditions is the most common risk factor for gallbladder cancer?

A. Cholelithiasis
B. Coledochal cysts
C. Primary sclerosing cholangitis
D. Liver fluke infestation

Correct answer is A. RATIONALE: Approximately 75% to 98% of patients with gallbladder


cancer have cholelithiasis.

248. What is the approximate D0 for oocytes?

A. 0.1 Gy
B. 0.3 Gy
C. 0.5 Gy
D. 0.7 Gy

Correct answer is A. RATIONALE: Oocytes are extremely radiosensitive, and like


lymphocytes, will undergo apoptosis after exposure to approximately 0.12 Gy (12 cGy).

70 American College of Radiology


249. For a Compton scattering event, the difference in wavelength between the incident and
scattered photon is:

A. independent of the electron mass.


B. independent of the initial photon energy.
C. a function of the incident photon energy.
D. inversely proportional to the photon scattering angle.

Correct answer is B. RATIONALE: Requires knowledge of the Compton process.

250. Which of the following chemotherapy agents is most commonly used when
administering chemoradiation for bladder preservation?

A. Adriamycin
B. Melphalan
C. Cisplatin
D. 5-FU

Correct answer is C. RATIONALE: Cisplatin is most commonly used when administering


chemoradiation for bladder preservation.

251. According to GOG 123 (Keys), what is the relative risk of disease progression among
the women assigned to receive preoperative radiation therapy and chemotherapy
compared to preoperative radiation therapy alone for bulky stage IB cervical cancer?

A. 0.26
B. 0.35
C. 0.51
D. 0.99

Correct answer is C. RATIONALE: The relative risk of recurrence in the combined-therapy


group, compared with the group given radiation therapy alone, was 0.51 (95% confidence
interval). Reference: New England Journal of Medicine (NEJM). 1999 (April 15);
340(15):1154-1161.

252. Multidrug resistance in tumor cells is often attributed to extrusion of drugs mediated by
overexpression of which of the following proteins?

A. P-glycoprotein
B. Bcl-2
C. BAX
D. TP53

Correct answer is A. RATIONALE: P-glycoprotein is a transmembrane protein product of the


multiple drug resistance (mdr) gene.

Radiation Oncology In-Training Exam 2007 71


253. The biological half-life of a systemically distributed radionuclide is 8 days, and the
physical half-life is 3 days. What is the effective decay constant?

A. 0.23 day-1
B. 0.32 day-1
C. 0.46 day-1
D. 0.66 day-1

Correct answer is B. RATIONALE: Requires knowledge of how to combine effects of physical


and biological decay.

254. Which of the following statements about temporal bone chemodectomas is true?

A. They are derived from neural crest tissue of the second branchial arch associated
with either cranial nerve IX or X.
B. Microscopic evaluation of these tumors reveals a high level of disorganization of
cellular alignment.
C. The majority occur in the inferior tympanic canaliculus.
D. They are very sensitive to chemotherapy.

Correct answer is A. RATIONALE: Chemodectomas look histologically benign and are


sensitive to radiation therapy, but not to chemotherapy. Only 20% of temporal bone
chemodectomas occur in the inferior tympanic canaliculus.

255. Which of the following conditions does NOT belong in the paraneoplastic category of
immune-mediated neurological syndromes in small cell lung cancer?

A. Eaton-Lambert syndrome
B. Cerebellar degeneration
C. Hypernatremia
D. Retinopathy

Correct answer is C. Reference: Gandhi, L, Johnson, BE. Journal of the National


Comprehensive Cancer Network (JNCCN). 2006;4:631-638.

256. What is the proportion of indirect versus direct damage in cells irradiated with
linear-accelerator-produced x-rays?

A. 70% indirect, 30% direct


B. 50% direct, 50% indirect
C. 40% indirect, 60% direct
D. 10% direct, 90% indirect

Correct answer is A. RATIONALE: Indirect damage secondary to the ionization of water


accounts for approximately two thirds of the total DNA damage produced in cells irradiated with
low-LET X- rays.

72 American College of Radiology


257. Which of the following is defined as a medical event by the Nuclear Regulatory
Commission (NRC)?

A. Total dose in 1 week from a linear accelerator that exceeds 20% of the weekly total
prescribed dose
B. Total dose from a low-dose-rate (LDR) procedure that exceeds 20% of the
prescribed dose
C. Dose in 1 week from a 60Co teletherapy machine that exceeds 15% of the weekly
total prescribed dose
D. Dose from a Gamma Knife stereotactic machine that exceeds 15% of the prescribed
dose

Correct answer is B. RATIONALE: Requires knowledge of NRC regulations.

258. Which of the following lymph node chains is most commonly involved in patients who
present with Hodgkin lymphoma?

A. Inguinal
B. Paraaortic
C. Mediastinal
D. Cervical

Correct answer is D. RATIONALE: Approximately 60% to 80% of patients with Hodgkin


lymphoma present with cervical/supraclavicular adenopathy. Approximately 50% of patients
have mediastinal lymph node involvement.

259. Which of the following radiation doses is standard definitive radiation therapy for an
unresectable Ewing sarcoma?

A. 45.0 Gy
B. 55.8 Gy
C. 64.8 Gy
D. 70.2 Gy

Correct answer is B. RATIONALE: The standard radiation dose is 55.8 Gy. Doses >60 Gy
have been associated with high rates of secondary malignancy.

260. Which of the following cell cycle phases is most radiosensitive?

A. M
B. S
C. G1
D. G0

Correct answer is A. RATIONALE: Cells synchronized in the G2- or M-cell cycle phases by
physical (mitotic shake off) or chemical means have been consistently shown to be more
clonogenically radiosensitive than other cell cycle phases.

Radiation Oncology In-Training Exam 2007 73


261. What is the primary source of contrast in megavoltage electronic portal images?

A. Pair production
B. Compton scattering
C. Photoelectric effect
D. Bremsstrahlung

Correct answer is B. RATIONALE: Compton scattering is the primary source of contrast with
portal imaging. The photoelectric effect is the primary source of contrast for diagnostic plain
films.

262. Which of the following statements about mesenchymal chondrosarcoma is true?

A. It occurs most commonly in the flat bones.


B. It occurs most commonly in elderly patients.
C. It has a better prognosis than most other forms of chondrosarcoma.
D. It has a monophasic histologic pattern.

Correct answer is A. RATIONALE: Mesenchymal chondrosarcomas are more aggressive and


have a worse prognosis than the more common subtypes. They most commonly occur in the
spine or non-pelvic flat bones.

263. Which of the following statements about level III involvement of the axillary lymph nodes
in a patient with breast cancer is true?

A. This region is immediately lateral to the infraclavicular fossa.


B. The region is superomedial to the pectoralis minor muscle and lateral to the
clavicular bone.
C. The lymph nodes are commonly removed by standard axillary lymph node
dissection.
D. Metastases to this region would be classified as stage N3b disease.

Correct answer is B. RATIONALE: The AJCC Cancer Staging Manual describes level III lymph
nodes as those in the axillary apex. This includes all lymph nodes medial to the medial margin
of the pectoralis minor muscle, including those designated as apical. Standard axillary lymph
node dissection includes level I and II lymph nodes. Stage N3b disease designates axillary
lymph node involvement (at any level) in association with ipsilateral IMC node involvement.
Level III lymph nodes are at the superior clinical border of the infraclavicular fossa.

264. The gastrointestinal syndrome is linked to compromised intestinal function secondary to


disruption of the normal villus architecture. What causes the changes in villi following
irradiation?

A. Killing of crypt cells


B. Migration of Goblet cells
C. Prolonged cell cycle arrest
D. Damage to villus vasculature

Correct answer is A. RATIONALE: Killing of crypt stem cells that resupply mature epithelial
cells to line the surfaces of the villi drives changes in villus structure, and ultimately, intestinal
function.

74 American College of Radiology


265. The American Association of Physicists in Medicine (AAPM) Task Group No. 43 (TG-43)
dose calculation formalism can be applied to which of the following types of treatment?

A. 90Y microspheres
B. 131I thyroid therapy
C. 137Cs Fletcher-Suit
D. High-dose-rate (HDR) endobronchial therapy

THIS ITEM WAS NOT SCORED. RATIONALE: AAPM TG43 was designed for small
encapsulated sources. 137Cs uses more elongated sources and may not be as applicable as
HDR therapy. 90Y and 131I are internally distributed throughout tissue and require a different
formalism.

266. Which of the following statements about ependymomas is true?

A. Patient survival is dependent on the degree of tumor resection.


B. They most commonly occur in patients older than 21 years of age.
C. Approximately 30% of patients with ependymomas have cerebrospinal fluid
dissemination.
D. Postoperative radiation therapy has no impact on local control in completely
resected tumors.

Correct answer is A. RATIONALE: Ependymoma most commonly affects children younger


than 5 years of age. The risk of cerebrospinal fluid (CSF) spread is approximately 5% to 10%.
Completeness of tumor resection affects patient survival, and the postoperative radiation
therapy dose impacts disease control (see Perez textbook).

267. Which of the following statements about patients with primary sclerosing cholangitis
(PSC) and cholangiocarcinoma is true?

A. Hepatobiliary cancer will develop in 25% to 30% of patients with PSC.


B. Cholangiocarcinoma tends to occur at an older age in patients who have PSC.
C. Approximately one third of cholangiocarcinomas occur within 10 years of diagnosis
of PSC.
D. Patients who have both conditions often have a worse prognosis.

Correct answer is D. RATIONALE: Patients with primary sclerosing cholangitis and


cholangiocarcinoma have a poor prognosis, because they often have multifocal cancer and
underlying liver dysfunction.

268. What percent of patients with bladder cancer invading the muscles will achieve a
complete response to induction chemoradiation administered for bladder preservation?

A. 30%
B. 40%
C. 55%
D. 65%

Correct answer is D. RATIONALE: About two thirds of patients who enter bladder preservation
protocols achieve complete response to induction therapy. Reference: Urology, 2002 (July);
60(1):62-7.

Radiation Oncology In-Training Exam 2007 75


269. Which of the following does NOT affect the response of the developing embryo or fetus
to ionizing radiation?

A. Stage of gestation
B. Body mass
C. Dose rate
D. Dose

Correct answer is B. RATIONALE: The stage of gestation, dose rate, and total dose are the
principal determinants of pathogenic radiation effects on the embryo or fetus.

270. Which of the following types of radiation is detected in PET imaging?

A. 140-keV photons
B. 0.5-MeV photons
C. 0.5-MeV electrons
D. Positrons

Correct answer is B. RATIONALE: The 0.5-MeV (511 keV) photons are produced from
annihilation and thus localize the positron. Lower-energy photons are from scatter and
decrease image quality by producing false coincidence events.

271. Based on the results of SWOG 8797 (Peters), which of the following pathologic findings
after a radical hysterectomy and pelvic lymph node dissection is an indication for
adjuvant chemotherapy with radiation therapy for patients with high-risk cancers of the
cervix?

A. Tumor size >5 cm


B. Lymphovascular space invasion
C. Depth of invasion into the deep third of the cervical stroma
D. Microscopic involvement of the parametrium

Correct answer is D. RATIONALE: This study included patients who had positive pelvic lymph
nodes and/or margins and/or microscopic parametrial involvement. The study concluded that
adjuvant chemotherapy and radiation therapy significantly improved progression-free survival
and overall survival in this high-risk population. Reference: Peters, et al: Concurrent
chemotherapy and pelvic radiation therapy compared with pelvic radiation alone as adjuvant
therapy after radical surgery in high-risk early stage cancer of the cervix. Journal of Clinical
Oncology (JCO). 2000 (April);18(8):1606-1613.

272. Which of the following statements about the role of radiation therapy in the management
of paragangliomas is true?

A. Surgery followed by radiation therapy is the preferred treatment.


B. The radiation dose response increases above 45 Gy for tumor regression.
C. The local control rate after radiation doses of 45 Gy alone is commonly 90% to 95%.
D. Compared to radiation therapy, surgery offers improved control and disease-specific
survival rates.

Correct answer is C. RATIONALE: Surgery or radiation therapy can be used to treat


paragangliomas. A radiation dose of 45 Gy is sufficient to achieve excellent local control
(>90%).

76 American College of Radiology


273. According to the VA Lung Cancer Group's classification, which of the following sites of
lymphadenopathy is associated with extensive-stage small cell lung cancer?

A. Ipsilateral hilar lymph nodes


B. Ipsilateral supraclavicular lymph nodes
C. Contralateral supraclavicular lymph nodes
D. Contralateral mediastinal lymph nodes

Correct answer is C. Reference: Journal of the National Comprehensive Cancer Network


(JNCCN). 2006;4:615.

274. Which of the following physical or biological features best distinguishes x-rays from
gamma rays?

A. X-rays are more penetrating in tissue.


B. X-rays are produced outside the atomic nucleus.
C. Gamma rays are less biologically effective.
D. Gamma rays produce less scatter in tissue.

Correct answer is B. RATIONALE: The physical source of these radiations is their defining
feature.

275. Doubling the atomic number has what effect on attenuation probability for photoelectric
interactions?

A. Decrease by a factor of 8
B. Decrease by a factor of 2
C. Increase by a factor of 8
D. Increase by a factor of 2

Correct answer is C. RATIONALE: Attenuation is proportional to Z^3/E^3 for photoelectric


interactions.

276. Which of the following factors increases a patient's risk for the development of
leukemia?

A. Occupational exposure to benzene


B. Occupational exposure to aluminum
C. Use of mitomycin C
D. Exposure to HPV

Correct answer is A. RATIONALE: Risk factors for leukemia include radiation exposure/atomic
bomb experience, smoking, and benzene exposure. There has been no established relationship
to viral exposure with HPV, mitomycin C, aluminum, or thalidomide.

Radiation Oncology In-Training Exam 2007 77


277. Which of the following genetic abnormalities is most commonly associated with Ewing
sarcoma?

A. t(8;14)
B. t(11;22)
C. 1p/19q gene deletion
D. Loss of heterozygosity (LOH) of the retinoblastoma gene

Correct answer is B. RATIONALE: t(11;22) is the classic translocation in Ewing sarcoma. The
translocation t(8;14) is most commonly seen in Burkitt lymphoma; LOH-RB in retinoblastoma;
and 1p/19q gene deletion in oligodendrogliomas.

278. Which of the following proteins plays a key role in homologous recombination?

A. Ku80
B. Rad51C
C. Telomerase
D. XPA

Correct answer is B. RATIONALE: Rad51C is part of a multi-protein complex that participates


in DNA DSB repair by homologous recombination in mammalian cells.

279. Which of the following SI units is used to express integral dose?

A. Gray
B. Sievert
C. Joule
D. Watt

Correct answer is C. RATIONALE: Requires an understanding of how integral dose is


determined.

280. The ages of two populations are sampled. One population has a mean age of 40 years
and a standard deviation of 3 years. The other population has a mean age of 30 years
and a standard deviation of 4 years. What is the likelihood that these two populations
are the same in age?

A. <0.05%
B. <5%
C. >75%
D. >95%

Correct answer is B. RATIONALE: This is a typical t test. The null hypothesis that
population A is the same as population B is rejected at p<0.05, as the means are different by at
least 2 standard deviations. The separation would have to be far greater to achieve 0.05%, or
p<0.0005.

78 American College of Radiology


281. Which of the following statements about the diagnosis of inflammatory breast cancer is
true?

A. There may not be a discrete palpable mass at presentation.


B. At presentation, a patient may report having a longstanding breast mass that
developed skin changes over time.
C. A biopsy is not required, since the diagnosis is clinically established.
D. Dermal lymph node involvement must be demonstrated.

Correct answer is A. RATIONALE: Inflammatory carcinoma is a clinicopathologic entity


characterized by diffuse erythema and edema (peau d’orange) of the breast, often without an
underlying palpable mass. These clinical findings should involve the majority of the skin of the
breast. Classically, the skin changes arise quickly in the affected breast. Thus, the term
“inflammatory carcinoma” should not be applied to a patient with neglected locally advanced
cancer of the breast presenting late in the course of the disease. On imaging, there may be a
detectable mass and characteristic thickening of the skin over the breast. This clinical
presentation is due to tumor emboli within dermal lymphatics, which may or may not be
apparent on skin biopsy. The tumor of inflammatory carcinoma is classified T4d. It is important
to remember that inflammatory carcinoma is primarily a clinical diagnosis. Involvement of the
dermal lymphatics alone does not indicate inflammatory carcinoma in the absence of clinical
findings. In addition to the clinical picture, however, a biopsy is still necessary to demonstrate
cancer either within the dermal lymphatics or in the breast parenchyma itself. Reference: AJCC
Cancer Staging Manual, 6th edition, 2002; pp 225-226.

282. For a population of 100 cells with a D0 of 2 Gy, how many cells on average would
survive a single dose of 2 Gy (based on use of Poisson statistics)?

A. 00
B. 37
C. 50
D. 63

Correct answer is B. RATIONALE: This question requires an understanding of target theory


and Poisson statistics. It illustrates the random nature of cell killing by radiation.

283. Which of the following statements about primary barriers for shielding calculations is
true?

A. They have a use factor of 0.5.


B. They have an average patient attenuation factor.
C. They have an occupancy factor of 1 for controlled access areas.
D. They are used only for controlled access areas.

Correct answer is C. RATIONALE: This item requires knowledge of shielding variables.

Radiation Oncology In-Training Exam 2007 79


284. Which of the following prognostic factors is included in the Radiation Therapy Oncology
Group (RTOG) recursive partitioning analysis (RPA)?

A. Tumor size
B. Tumor stage
C. Mental status
D. Extent of resection

Correct answer is C. RATIONALE: RTOG recursive partitioning analysis included the following
prognostic factors: patient's age, Karnofsky performance status, duration of neurologic
symptoms, mental status.

285. Monte Carlo models are applied to statistical studies by:

A. comparing means and standard deviations.


B. extrapolating from limited samples to predict endpoints.
C. calculating radiation dose through generating transport histories.
D. randomly sampling a modeled population multiple times.

THIS ITEM WAS NOT SCORED.

286. Which of the following cell cycle phase transitions is most affected when the TP53 gene
is lost or mutated?

A. G0 → G1
B. G1 → S
C. G2 → M
D. S → G2

Correct answer is B. RATIONALE: Early work by Michael Kastan on the G1→S cell cycle
phase transition has unambiguously linked the TP53 gene to this process.

287. What is the daughter radionuclide for a 90Sr eye applicator?


89
A. Sr
91
B. Sr
89
C. Y
90
D. Y

Correct answer is D. RATIONALE: Requires knowledge of 90Sr decay.

288. Bladder preservation therapy with chemoradiation would be most appropriate for bladder
cancer associated with:

A. hydronephrosis.
B. multifocal carcinoma in situ.
C. invasion of the subepithelial connective tissue.
D. invasion of the outer half of the bladder wall.

Correct answer is D. RATIONALE: Bladder preservation therapy is best suited for muscle-
invasive bladder cancer without hydronephrosis and multifocal carcinoma in situ.

80 American College of Radiology


289. Which of the following statements about definitive radiation therapy for squamous cell
carcinoma of the vagina is true?

A. After 45 Gy of external-beam radiation therapy, a pelvic examination is performed to


determine the method of further radiation therapy delivery.
B. If the tumor involves the upper third of the vagina, the medial inguinofemoral lymph
nodes should be treated.
C. The inferior border of the fields is placed 1 cm below the most distal extent of the
disease.
D. Radiation therapy achieves pelvic disease control rates of 80% to 90% for patients
with stage III to stage IVA disease.

Correct answer is A. RATIONALE: Answer B would be true if the tumor involved the lower third
of the vagina. Answer C places the inferior border too close to the gross disease. Answer D is
false, but would be true if stage I-II disease. Reference: Steven J. Frank, Anuja Jhingran,
Charles Levenback, and Patricia J. Eifel. Definitive radiation therapy for squamous cell
carcinoma of the vagina. International Journal of Radiation Oncology, Biology, Physics.
2005 (May);62(1):138-147.

290. Which of the following molecular mechanisms is specifically implicated in the


oncogenesis of cervical cancer associated with prior HPV infection?

A. The E7 protein expression inactivates p53.


B. The E7 protein expression inhibits Rb function.
C. The E6 protein expression suppresses G2-M cell cycle arrest.
D. The E6 protein expression activates c-myc.

Correct answer is B. RATIONALE: Some HPV viral genes have specific effects on cell cycle
control that directly impact tumor initiation. Specifically, the E7 protein binds to pRB leading to
its degradation, which in turn releases the E2F transcription factor to induce cellular DNA
synthesis and progression into the S cell cycle phase.

291. For alpha particle decay, the neutron/proton (N/P) ratio of the daughter nuclide
compared to that of the parent nuclide:

A. increases.
B. decreases.
C. remains constant.
D. has a value closer to 1.

THIS ITEM WAS PARTIALLY SCORED (BOTH OPTIONS A & C SCORED CORRECT).
RATIONALE: Requires knowledge of alpha particle decay and basic math.

Radiation Oncology In-Training Exam 2007 81


292. Which of the following neurological complications is most common after surgical
resection of paragangliomas?

A. Superior laryngeal nerve paralysis


B. Spinal accessory nerve injury
C. Hypogossal nerve injury
D. Vagus nerve injury

Correct answer is D. RATIONALE: Injury to cranial nerves IX and X are the most common
neurological complications after surgical resection of paragangliomas.

293. Which of the following radiation doses is LEAST preferred for prophylactic cranial
irradiation administered to patients who have limited-stage small cell lung cancer and
who experience a complete response to chemotherapy for their lung lesions?

A. 24 Gy in 8 fractions
B. 25 Gy in 10 fractions
C. 30 Gy in 10 fractions
D. 30 Gy in 15 fractions

Correct answer is C. Reference: Journal of the National Comprehensive Cancer Network


(JNCCN). 2006;4:619.

294. What is the approximate relative biological effectiveness (RBE) of protons with energies
that are commonly used in radiation therapy?

A. 0.7
B. 1.2
C. 1.8
D. 2.6

Correct answer is B. RATIONALE: A relative biological effectiveness (RBE) of 1.2 would be


expected for proton beams with energies of approximately 160 MeV used for radiation therapy.

295. Which of the following uncertainties contributes most to the planning target volume
expansion?

A. Beam penumbra width


B. Random variation in patient position
C. Systematic variation in patient position
D. Laser calibration differences between the CT scanner and linear accelerator

Correct answer is C. RATIONALE: Laser calibration differences should be within 1 mm to


2 mm, much smaller than position variability. Random variation has significantly less impact (by
a factor of 3 or more) than systematic position error. Penumbra is not considered in planning
target volume (PTV) definition.

82 American College of Radiology


296. Which of the following factors is associated with chronic myelogenous leukemia?

A. t(5;17)
B. t(9;22)
C. Trisomy 8
D. Trisomy 21

Correct answer is B. RATIONALE: The chromosomal translocation t(9;22) is called the


"Philadelphia (Ph) chromosome," and translocation of the BCR and ABL genes are present in
approximately 90% of chronic myelogenous leukemia (CML) cases.

297. What is the most common site of metastatic disease in patients with Ewing sarcoma?

A. Lung
B. Bone
C. Bone marrow
D. Lymph nodes

Correct answer is A. RATIONALE: The most common site of distant spread in patients with
Ewing sarcoma is the lungs, followed closely by the bone.

298. Which of the following types of ionizing radiation is produced when low-energy (thermal)
neutrons bombard a boron-containing compound during boron-neutron capture therapy?

A. Protons
B. Photons
C. Alpha particles
D. High-energy neutrons

Correct answer is C. RATIONALE: During boron-neutron capture therapy, 10B, is irradiated


with low-energy neutrons (0.025 eV) to yield helium nuclei (alpha particles) and 7Li nuclei.

299. A point on a cumulative dose volume histogram represents the:

A. dose received by a given point in an organ.


B. volume of an organ that receives a given dose or less.
C. volume of an organ that receives a given dose or greater.
D. location of a given dose in an organ.

Correct answer is C. RATIONALE: By definition, each point on a cumulative dose volume


histogram (DVH) represents the definite integral of volume from that dose to the maximum dose
in the body.

Radiation Oncology In-Training Exam 2007 83


300. Which of the following statements about metastasis of breast cancer to the bone is true?

A. Metastasis to the epidural spine is most commonly associated with worsening back
pain in the supine position or with the Valsalva maneuver.
B. Compression of the epidural spinal cord associated with breast cancer most
commonly has a single site of vertebral metastasis.
C. Radicular pain in the thoracic spine is typically unilateral.
D. Pain from T12, L1 vertebral lesions is not associated with a referred pain pattern to
the bilateral iliac crests or to the bilateral sacroiliac joints.

Correct answer is A. Reference: Harris, JR. Diseases of the Breast, 3rd edition, Lippincott,
Williams and Wilkins, 2004; p. 1220.

301. Which of the following radiation treatment sites would be most appropriate for a patient
with breast cancer who undergoes postoperative irradiation after successful wide
excision for a localized chest wall recurrence 2 years after mastectomy and
chemotherapy?

A. Chest wall only


B. Chest wall and draining lymph nodes
C. Excision scar and a 5-cm margin
D. No radiation is required if the excision reveals negative margins.

Correct answer is B. RATIONALE: Higher rates of locoregional tumor control are reported
when radiation field design includes the chest wall and supraclavicular areas, +/- the IMC or
axillary lymph nodes. Reference: Perez and Brady: Locally Advanced and Recurrent Breast
Cancer, 2004; pp 1529-1532.

302. Which of the following factors best explains why hypoxic cells are more resistant to the
cytotoxic effects of ionizing radiation?

A. Up-regulation of HIF-1α
B. Down-regulation of TP53
C. Decreased cell proliferation
D. Decreased free-radical damage

Correct answer is D. RATIONALE: A relative lack of free oxygen reduces the probability of
fixation of free-radical damage in irradiated DNA, and therefore, accounts for the reduced
cytotoxicity under hypoxic conditions.

303. An incident 2-MeV photon undergoes Compton scatter. The resulting electron receives
0.5 MeV of kinetic energy and generates two bremsstrahlung photons of 100 keV and
30 keV as it travels. What is the collision kerma of this event?

A. 0000.5 MeV
B. 0002 MeV
C. 0370 keV
D. 1870 keV

Correct answer is C. RATIONALE: Kc (collision kerma) is the energy transferred to the


medium, minus that lost to bremsstrahlung.

84 American College of Radiology


304. Which of the following statements about craniopharyngiomas is true?

A. Complete surgical resection is possible in approximately 10% to 15% of patients.


B. Surgery is rarely associated with visual and endocrine complications.
C. Conformal external-beam radiation therapy of 40 Gy delivered to both cystic and
solid components with a 0.5-cm margin will achieve local control.
D. Intracavitary diluted β-emitter isotopes can be used to treat cystic lesions with up to
200 Gy prescribed to the cyst wall.

Correct answer is D. RATIONALE: The recommended dose for external-beam radiation


therapy is about 54 to 55 Gy. P-32, a beta-emitter, can be used to treat monocystic lesions.

305. Which of the following statements about the fibrolamellar variant of hepatocellular cancer
is true?

A. It affects men and women equally.


B. It occurs more frequently in elderly persons.
C. It is commonly associated with cirrhosis of the liver.
D. Patients are less likely to present with positive lymph nodes.

THIS ITEM WAS PARTIALLY SCORED (BOTH OPTIONS A & D SCORED CORRECT).
RATIONALE: No clear sex predominance is reported in fibrolamellar hepatocellular cancer.

306. Which of the following repair pathways rejoins DNA double-strand breaks?

A. Mismatch repair
B. Nucleotide excision repair
C. Non-homologous end joining
D. Base excision repair

Correct answer is C. RATIONALE: Only two pathways can rejoin DNA DSB: non-homologous
end joining (NHEJ) and homologous recombination (HR).

307. Which of the following pairs of processes results in the production of characteristic
x-rays?

A. Alpha decay and electron capture


B. Positron decay and isomeric transition
C. Negatron decay and internal conversion
D. Electron capture and internal conversion

Correct answer is D. RATIONALE: Requires an understanding of radioactive decay pathways.

308. Which of the following statements about penile cancer is true?

A. Greater than 90% have squamous cell histology.


B. Clinically positive lymph nodes are pathologically involved <20% of the time.
C. Tumors most commonly originate from the shaft of the penis.
D. First-echelon lymph nodes include the periaortic lymph nodes.

Correct answer is A. RATIONALE: Squamous cell carcinoma is the predominant type of


cancer in the penis.

Radiation Oncology In-Training Exam 2007 85


309. What is the 5-year disease-specific survival rate for stage I squamous cell carcinoma of
the vagina treated with definitive radiation therapy?

A. 95%
B. 85%
C. 75%
D. 50%

Correct answer is B. Reference: Steven J. Frank, Anuja Jhingran, Charles Levenback, and
Patricia J. Eifel: Definitive radiation therapy for squamous cell carcinoma of the vagina.
International Journal of Radiation Oncology, Biology, Physics. 2005 (May);62(1):138-147.

310. Which of the following represents the most appropriate application of the linear quadratic
model of cell killing?

A. Predicting tumor-control doses


B. Predicting normal-tissue tolerance doses
C. Comparing different fractionation schedules
D. Optimizing beam energy

Correct answer is C. RATIONALE: The linear quadratic equation is best viewed as a guide to
the likely fractionation sensitivity of different tissues; and as such, it can be used to compare
different treatment schedules for approximate biological equivalence (or lack thereof). It should
not be used to predict or assess treatment outcome in individual patients.

311. Which of the following units was used for calibration of the brachytherapy source in the
report by the American Association of Physicists in Medicine (AAPM) Task Group No. 43
(TG-43)?

A. Kerma rate
B. Collision kerma
C. Air kerma strength
D. Apparent activity

Correct answer is C. RATIONALE: Requires knowledge of AAPM Task Group No. 43 (TG-43)
report.

312. Which of the following lymph nodes is at the highest risk for regional metastasis from
skin cancer of the forehead?

A. Parotid
B. Suboccipital
C. Submental
D. Facial

Correct answer is A. RATIONALE: Knowledge of the cutaneous lymphatic drainage of the


head and neck is important for determining the lymph node groups at risk for regional
metastasis. Tumors of the scalp and forehead anterior to a coronal plane through the ear
canals spread preferentially to the parotid and upper jugular lymphatic chain.

86 American College of Radiology


313. A 62-year-old man with limited-stage small cell lung cancer and proximal muscular
weakness is most likely to have which of the following conditions?

A. SIADH
B. Myasthenia gravis
C. Cerebellar degeneration
D. Eaton-Lambert syndrome

Correct answer is D. RATIONALE: This is a typical presentation of Eaton-Lambert Myasthenic


syndrome seen in about 1% to 2% of patients with small cell lung cancer. EMG and antibody
testing to P/Q VGCC are diagnostic. Reference: Darnell, RB, Posner, JB. Paraneoplastic
syndromes affecting the nervous system, Seminars in Oncology, 2006,33:270-298.

314. Which of the following statements most accurately describes the biological basis for
combining heat and radiation for the treatment of malignant disease?

A. Heat suppresses radiation-induced checkpoint activation


B. Heat is preferentially toxic to tumor cells.
C. Heat inhibits DNA repair
D. Heat triggers apoptosis

Correct answer is C. RATIONALE: Heat affects the stability of many nuclear proteins including
those involved in DNA repair. This is thought to be the basis of thermal sensitization.
Reference: Kampinga. International Journal of Hyperthermia. 2006 (May);22(3):191-6.

315. Which of the following qualities causes film to be useful for analyzing relative dose
distributions?

A. An effective Z value equal to water


B. No background corrections required
C. Insensitivity to film-processing conditions
D. Small grain size

Correct answer is D. RATIONALE: Requires knowledge of film dosimetry.

316. Which of the following types of leukemia has the highest risk of CNS involvement?

A. Acute lymphocytic
B. Acute myelogenous
C. Chronic myelogenous
D. Chronic lymphocytic

Correct answer is A. RATIONALE: The eye/ocular region, testicles, and central nervous
system (CNS) are common sanctuary sites in acute lymphocytic leukemia (ALL), with the CNS
being the most common. Because of this, CNS prophylaxis is often employed.

Radiation Oncology In-Training Exam 2007 87


317. A 3-year-old girl has Wilms tumor with a favorable histology and a positive surgical
margin. CT scan of the chest does not reveal any pulmonary nodules. What cancer
stage is most appropriate for the patient's condition?

A. Stage I
B. Stage II
C. Stage III
D. Stage IV

Correct answer is C. RATIONALE: Cancer stage III is associated with this patient’s condition
because of the positive surgical margin.

318. Which of the following types of chromosomal aberrations is most likely to be involved in
carcinogenesis?

A. Rings
B. Dicentrics
C. Anaphase bridges
D. Translocations

Correct answer is D. RATIONALE: Rings, anaphase bridges, and dicentrics are generally
considered to be lethal types of chromosomal aberrations. Translocations (and small deletions)
are often compatible with continued viability but may lead to cancer.

319. According to the Nuclear Regulatory Commission (NRC), a written directive for a
brachytherapy procedure requires at least which of the following two pieces of
information?

A. Source type and the source serial number


B. Both the authorized user and radiation safety officer signatures
C. Patient name and the source serial number
D. Patient name and number of sources

Correct answer is D. RATIONALE: Requires knowledge of written directive requirements.

320. Which of the following types of cancer is most frequently associated with isolated
hepatic metastases at presentation?

A. Colorectal
B. Esophageal
C. Gastric
D. Lung

Correct answer is A. RATIONALE: Colorectal cancer is the most frequent type of malignancy
that is associated with isolated hepatic metastasis at presentation.

88 American College of Radiology


321. Which of the following findings is associated with a better outcome for patients who have
a local recurrence of breast cancer after breast-conserving surgery?

A. Metaplastic histology
B. A disease-free interval of <2 years
C. Recurrent DCIS in a different quadrant of the breast
D. Concurrent involvement of the supraclavicular lymph nodes

Correct answer is C. RATIONALE: Noninvasive breast cancer in a different quadrant of the


breast is suggestive of a new primary lesion. The cure rate with salvage mastectomy is nearly
100%. Other options are known to be associated with a poor prognosis after recurrence.

322. What percent of patients with hepatocellular cancer are candidates for curative
resection?

A. 75%
B. 50%
C. 20%
D. 05%

Correct answer is C. RATIONALE: In high-incidence regions of the world, only 10% to 15% of
newly diagnosed patients are candidates for standard resection; whereas in low incidence
areas, between 15% and 30% of patients are potential candidates for resection.

323. Which of the following sites is most commonly associated with cancer of the urethra in
men?

A. Fossa navicularis
B. Bulbomembranous urethra
C. Prostatic urethra
D. Penile urethra

Correct answer is B. RATIONALE: Urethral tumors in men occur predominantly in the


bulbomembranous urethra.

324. What is the FIGO stage of a 3-cm vaginal mass with paravaginal extension and a single
metastatic inguinal lymph node?

A. II
B. III
C. IVA
D. IVB

Correct answer is D. RATIONALE: Based on the AJCC Cancer Staging Handbook (6th
edition), N1 disease is stage IVB using the FIGO staging system (or stage III using the TNM
staging system).

Radiation Oncology In-Training Exam 2007 89


325. An excisional biopsy is NOT appropriate for a suspected melanoma in which of the
following sites?

A. Palmar surface of the hand


B. Dorsal surface of the hand
C. Forearm skin
D. Shoulder

Correct answer is A. RATIONALE: Excisional biopsy may be inappropriate for certain sites
such as the face, palmar surface of the hand, sole of the foot, or distal digit. In these instances,
a full-thickness incisional or punch biopsy of the clinically thickest portion of the lesion is
recommended, rather than a shave biopsy. It should provide adequate microstaging, without
interfering with definitive local therapy. (Refer to current NCCN guidelines.)

326. The 5-year survival for patients who have undergone radiation therapy for inoperable
stage III invasive thymoma is approximately:

A. 20%.
B. 50%.
C. 70%.
D. 90%.

Correct answer is B. RATIONALE: NCI, PDQ, Invasive thymoma and thymic carcinoma,
inoperable stage III and III invasive thymoma, Level of evidence, 3iiiDiii.

327. Which of the following types of leukemia is associated with myeloperoxidase activity and
cell markers CD13 and CD33?

A. Acute lymphocytic
B. Acute myelogenous
C. Chronic myelogenous
D. Chronic lymphocytic

Correct answer is B. RATIONALE: Myeloperoxidase activity and cell markers CD13 and CD33
are related to acute myelogenous leukemia (AML). Cell markers CD2, CD5, and CD7 are
related to T-cell acute lymphocytic leukemia (ALL), and CD10, CD19, and CD22 are related to
B-cell ALL.

328. Which of the following assays is commonly used for measurement of radiation-induced
DNA double-strand breaks?

A. Alkaline elution
B. Northern blotting
C. High-pressure liquid chromatography (HPLC)
D. Pulsed-field gel electrophoresis

Correct answer is D. Alkaline elution is used for detection of single-strand breaks, while HPLC
is used to detect base damage. Northern blotting is used for analysis of RNA.

90 American College of Radiology


329. What adjustment to the gantry angle would be required to make each pair of symmetric,
10-cm-wide opposed fields tangential at their posterior borders?

A. 10° anterior
B. 10° posterior
C. 03° anterior
D. 03° posterior

Correct answer is D. RATIONALE: Adjustment equals arc tangent (half field width/source axis
distance), which is 2.9°. Beams must be angled posteriorly to make the posterior edge match.

330. Which of the following statements about radiation dose to the contralateral breast is
true?

A. Conventional medial/lateral tangential breast irradiation using 6-MV photons with a


physical wedge technique delivers 3% to 6% of the prescribed dose to the
contralateral breast.
B. A lower radiation dose is delivered to the contralateral breast with a physical wedge
technique than with a multileaf collimated intensity modulation technique.
C. A lower radiation dose is delivered to the contralateral breast if a medial wedge is
omitted and lateral tangential compensation is made with a physical wedge rather
than with a virtual wedge.
D. Irradiation using 6-MV photons without any compensation will deliver a 4% radiation
dose to the contralateral breast.

Correct answer is A. RATIONALE: Dose to the contralateral breast with 6-MV photons without
any compensation is 2%. Contralateral breast doses at 4.5 cm from the medial tangential
border by treatment technique are:
Cerrobend half beam block + wedge 8%
Cerrobend half beam block 7%
Asymmetric jaw + wedge 3.5%
Asymmetric jaw + block + wedge 3.5%
Asymmetric jaw 2.3%
Asymmetric jaw + block 2.5%
Non-diverging posterior border + wedge 3.5%
Non-diverging posterior border 3.5%
Virtual/dynamic wedge (med & lat) 2.3%
Virtual/dynamic wedge (lat only) 2.3%
Multileaf collimation (med & lat) 2.1%
Reference: Change, SX, et al. A comparison of different intensity modulation treatment
techniques for tangential breast irradiation. International Journal of Radiation Oncology, Biology,
Physics. 1999;45(5):1305-1314. Kelly, CA, et al. Dose to the contralateral breast: a
comparison of four primary techniques. International Journal of Radiation Oncology, Biology,
Physics. 1996;34(3):727-732.

Radiation Oncology In-Training Exam 2007 91


331. Which of the following statements about the treatment of bone metastases secondary to
breast cancer is true?

A. Radiation therapy will achieve partial or complete pain relief within 4 weeks.
B. Single-fraction irradiation provides equivalent results to protracted radiation
treatment schedules for pain control durability and reduced risk of subsequent
fracture.
C. Metastatic sites with extraosseous bone involvement should be treated with 153Sm
only, because localized external-beam irradiation is not beneficial.
D. An 8-Gy dose of hemibody irradiation should be administered in a single fraction to
the upper, middle, or lower hemibody regions for treatment of painful bony
metastases.

Correct answer is A. RATIONALE: Partial relief may be observed within several days or 1 to 2
weeks of radiation therapy, but commonly pain responses will be best assessed at the 4-week
interval. Single-fraction radiation therapy is equally effective for pain relief; however, multiple
studies have shown that over time there are higher rates of retreatment, subsequent bone
fracture, and decreased recalcification seen in single fraction treatment arms. Samarium must
be used in conjunction with local radiation when there is extraosseous tumor extension, since
Samarium will have no effect on tumor activity at soft tissue sites. Hemibody irradiation can be
recommended for treatment of pain secondary to widespread metastatic disease.
Recommended doses are 6 Gy per fraction to the upper hemibody and 8 Gy fractions to the
middle and lower hemibody regions. Duration of response in 14 weeks, and acute toxicities are
better with these dose selections. Reference: Harris, JR. Diseases of the Breast. 3rd edition.
Lippincott, Williams and Wilkins, 2004; pp. 1309-1320.

332. A Cox analysis is an alternative to:

A. linear regression when the responses are not normally distributed.


B. accelerated failure time analysis when the distribution of failure times can be
well-characterized by a failure distribution with a single parameter for scale.
C. Kaplan-Meier analysis when there is more than one predictor and/or the predictor is
a continuous variable.
D. Wilcoxon test when the within-strata hazard functions are proportional.

THIS ITEM WAS NOT SCORED.

333. Which stage corresponds to a rectal cancer invading through the muscularis propria with
metastasis to the perirectal lymph nodes?

A. T3N3
B. T3N2
C. T2N3
D. T2N1

THIS ITEM WAS NOT SCORED.

92 American College of Radiology


334. Stage T2 carcinoma of the urethra in men invades the:

A. corpus cavernosum.
B. corpus spongiosum.
C. subepithelial connective tissue.
D. bladder neck.

Correct answer is B. RATIONALE: Stage T2 carcinoma involves tumor invading any of the
following sites: corpus spongiosum, prostate, or periurethral muscle.

335. Which of the following statements about female urethral carcinoma is true?

A. A T2 tumor is associated with anterior vaginal invasion.


B. The T stage is affected by the size of the primary lesion.
C. The size of regional lymph node metastases affects the prognosis.
D. Two left-sided 2-cm metastatic inguinal lymph nodes have a better prognosis than a
single 2-cm lymph node in both inguinal regions.

Correct answer is C. RATIONALE: Regional lymph nodes important for staging of urethral
cancers include the superficial or deep inguinals; common, internal or external iliac nodes; and
presacral, sacral, and pelvic groups. The significance of regional lymph node metastasis in
staging urethral cancer lies in the number and size, not whether metastasis is unilateral or
bilateral. T size is established by the depth of the invasion. Reference: AJCC Cancer Staging
Handbook (6th edition), 2002.

336. Which of the following conditions places a patient at the highest risk for the development
of aggressive multi-focal squamous cell carcinoma of the skin?

A. Actinic keratosis
B. Chronic immunosuppression
C. Chronic arsenic exposure
D. Bowen's disease

Correct answer is B. RATIONALE: All of the above factors are known to increase squamous
cell carcinoma of the skin. Individuals who are chronically immunosuppressed have the highest
risk to develop multiple skin cancers, some of which behave in a very aggressive fashion.

337. Which of the following diagnostic tests is NOT included in the staging of small cell lung
cancer?

A. CT scan of the chest and upper abdomen


B. Brain imaging
C. Bone scan
D. PET scan

Correct answer is D. RATIONALE: A PET scan is considered a supplementation and not a


replacement to CT scanning, brain imaging, and bone scans. The use of PET scans for
diagnostic staging of small cell lung cancer is undergoing evaluation in clinical trials.

Radiation Oncology In-Training Exam 2007 93


338. Which of the following radiation dose ranges administered to the cranium would be most
appropriate for providing CNS prophylaxis to patients with acute lymphoblastic
leukemia?

A. 05 Gy to 10 Gy
B. 12 Gy to 18 Gy
C. 24 Gy to 30 Gy
D. 36 Gy to 40 Gy

Correct answer is B. RATIONALE: When CNS prophylaxis is provided in the management of


acute lymphoblastic leukemia (ALL), a radiation dose of 12 Gy to 18 Gy is typically
recommended. Intrathecal methotrexate is commonly used in adults.

339. Which of the following types of DNA damage would be predominant in cells after
irradiation with 1 Gy of x-rays?

A. Single-strand breaks
B. Double-strand breaks
C. Thymine dimers
D. Base damage

Correct answer is D. RATIONALE: Various authors have quoted the amount of damaged
bases per gray to be between 1,000 and 5,000, the number of single-strand breaks to be
approximately 1,000, and the number of double-strand breaks to be approximately 40 to 50 per
cell. Thymine dimers are typically induced by UV radiation.

340. Increasing the source-to-skin distance for a 6-MeV electron field from 100 cm to 110 cm
will have what effect on the dose to dmax?

A. Increase it by 10%
B. Increase it by 18%
C. Decrease it by 10%
D. Decrease it by 18%

Correct answer is D. RATIONALE: Inverse square law, with a minor adjustment as effective
source distance for electrons is generally less than 100 cm from isocenter (90 cm, in this
example).

341. Which of the following statements about clinical trials is true?

A. Unpublished trials should be included in a meta-analysis.


B. A phase III trial compares two experimental treatments in a randomized fashion.
C. In a phase III clinical trial, all patients who do not receive the intended treatment
should be excluded in the analysis.
D. Subgroup analysis is a reliable way to analyze a subgroup of patients with some
particular characteristics.

Correct answer is A. RATIONALE: All unpublished trials also should be included in


meta-analysis to minimize the effect of publication bias.

94 American College of Radiology


342. What is the typical single fraction hemibody dose that is used to treat bone metastasis?

A. 04 Gy
B. 06 Gy
C. 10 Gy
D. 30 Gy

Correct answer is B. RATIONALE: Single doses of 6 Gy or 8 Gy have been used to treat the
hemibody for palliation of skeletal metastases.

343. What percent of palpable inguinal nodes in patients with anal cancer are due to reactive
hyperplasia?

A. 10%
B. 25%
C. 50%
D. 66%

Correct answer is C. RATIONALE: Fifty percent of palpable inguinal lymph nodes in patients
with anal cancer are due to reactive hyperplasia.

344. Which of the following radiation therapy technique modifications will result in a lower
radiation dose to the contralateral breast?

A. A source-to-axis distance (SAD) instead of a source-to-surface distance (SSD)


technique
B. No medial wedge paired with a 30° lateral wedge compared with a 15° medial and
lateral wedge technique
C. A Cerrobend half-beam block instead of an asymmetric jaw for definition of the
posterior tangential border
D. A posterior nondiverging border (standard collimation) with a customized Cerrobend
block compared with an asymmetric jaw to define the posterior tangential border

Correct answer is B. RATIONALE: Fundamental understanding of how radiation technique


affects contralateral breast dose is necessary to reduce the potential late effects of treatment,
especially for younger patients. The principal factors affecting dose to the contralateral breast
are SAD vs. SSD technique, how the posterior tangential border is defined, use of beam
modifiers (physical wedge, virtual/dynamic wedges, compensating filter, intensity modulated
filters), and the clinical volume/symmetry of the contralateral breast itself. SSD technique will
increase scatter to the opposite breast relative to isocentric technique. Cerrobend half beam
block technique gives the highest contralateral breast dose, regardless of wedge application. A
medial wedge will always increase contralateral breast dose, regardless of any other technique
application, largely due to increased scatter as well as an increase in the number of monitor
units needed to deliver the correct dose to isocenter. Symmetrical jaws will give lowest doses to
the contralateral breast even when used with a customized cerrobend block. Non-diverging
posterior borders give a smaller dose compared with asymmetrical jaw and cerrobend block
techniques. Bhatnager describes an average of 11% superficial/skin dose to the opposite breast
with IMRT techniques. The amount of scatter was proportional to the breast size. References:
Bhatnagar, AK, et al. Does breast dose affect the scatter dose to the ipsilateral lung, heart, or
contralateral breast in primary breast irradiation using intensity-modulated radiation therapy
(IMRT)? American Journal of Clinical Oncology. 2006;29(1):80-84.

Radiation Oncology In-Training Exam 2007 95


(References continued for item 344.): Chang, SX, et al. A comparison of different intensity
modulation treatment techniques for tangential breast irradiation. International Journal of
Radiation Oncology, Biology, Physics. 1999;45(5):1305-1314. Kelly, CA, et al. Dose to the
contralateral breast: a comparison of four primary techniques. International Journal of Radiation
Oncology, Biology, Physics. 1996;34(3):727-732. Siddon, RL, et al. Three field technique for
breast irradiation using tangential field corner blocks. International Journal of Radiation
Oncology, Biology, Physics. 1983;9:583-588.

345. Which of the following types of ionizing radiation has similar biological properties to
protons?

A. Neutrinos
B. Alpha particles
C. Carbon ions
D. X-rays

Correct answer is D. RATIONALE: Protons and 250-kVp x-rays have a similar relative
biological effectiveness (RBE) and similar oxygen enhancement ratio (OER).

346. Which of the following statements about male urethral cancer is true?

A. Involvement of the lymph nodes is frequently bilateral.


B. Adenocarcinoma is the most common histological type.
C. Distal tumors tend to be more advanced than proximal tumors.
D. Thirty percent of clinically positive lymph nodes are pathologically positive.

Correct answer is A. RATIONALE: Lesions of the entire urethra or posterior urethra in women
and in the bulbomembranous urethra in men usually are associated with invasion and a high
incidence of pelvic lymph node metastases.

347. What is the stage of a squamous cell carcinoma of the female urethra that extends into
the bladder neck with a 2.5-cm left inguinal lymph node metastasis?

A. T2N2
B. T3N1
C. T3N2
D. T4N1

Correct answer is C. RATIONALE: Based on the AJCC Cancer Staging Handbook (6th edition,
2002), a stage T2 tumor invades the periurethral muscle. A stage T3 tumor invades the anterior
vagina and/or the bladder neck. A stage T4 tumor invades other adjacent organs. Stage N1
involves metastases to a single lymph node 2 cm or less in greatest dimension. Stage N2
involves metastases to a single lymph node more than 2 cm in greatest dimension or in multiple
lymph nodes.

96 American College of Radiology


348. Which of the following types of radiation is most responsible for the development of skin
cancer?

A. Cosmic rays
B. Long-wave ultraviolet (UVA) rays
C. Medium-wave ultraviolet (UVB) rays
D. Short-wave ultraviolet (UVC) rays

Correct answer is C. RATIONALE: The knowledge of the risk factors for skin cancers.
Medium-wave ultraviolet (UVB) rays are the principal carcinogen for the induction of skin
cancers. Long-wave ultraviolet (UVA) rays are primarily responsible for the sun's aging effects,
such as loss of skin elasticity. Short-wave ultraviolet (UVC) rays are a potent carcinogen, but
play a negligible role in the pathogenesis of skin cancers, because they are filtered out by the
atmospheric ozone layer.

349. Which of the following types of cancer is most commonly associated with myasthenia
gravis?

A. Small cell lung cancer


B. Non-small cell lung cancer
C. Mesothelioma
D. Thymoma

Correct answer is D. Reference: Engels, EA, Pfeiffer, RM. International Journal of Cancer.
2003;105(4):546-551.

350. During evaluation of tumor biopsy specimens, which of the following markers can be
used as a histochemical indicator of cell proliferation?

A. Ki-67
B. TP53
C. c-myc
D. Nuclear/cytoplasmic ratio

Correct answer is A. RATIONALE: Ki-67 is a marker of cells in the S cell cycle phase and is a
commonly used clinical indicator of cellular proliferation status.

351. If a typical lung region has an attenuation of 25% relative to water, then a CT scan of
this region should have what average Voxel value (in Hounsfield units)?

A. 0+25 HU
B. 0–25 HU
C. -250 HU
D. –750 HU

Correct answer is D. RATIONALE: HU = (attenuation of tissue – attenuation of


water)/(attenuation of water)*constant. The typical constant is ~1000. Thus, lung has
HU = (.25-1)*1000 = –750.

Radiation Oncology In-Training Exam 2007 97


352. Which of the following types of leukemia has the highest incidence rate?

A. Acute lymphocytic
B. Acute myelogenous
C. Chronic myelogenous
D. Chronic lymphocytic

Correct answer is D. RATIONALE: Chronic lymphocytic leukemia (CLL) is approximately twice


as common in men as in women and is more common than other leukemias.

353. What is the correct order of threshold doses, from lowest to highest, for the induction of
each of the three forms of the acute radiation syndrome (ARS)?

A. Cerebrovascular, Hematopoietic, Gastrointestinal


B. Gastrointestinal, Hematopoietic, Cerebrovascular
C. Hematopoietic, Gastrointestinal, Cerebrovascular
D. Hematopoietic, Cerebrovascular, Gastrointestinal

Correct answer is C. RATIONALE: The acute radiation syndrome is derived from studies on
animals (mice, rats) and humans that show a progression of organ failure based on total dose
administered. The underlying controlling factors are primarily the rate of stem cell turnover in
the first two syndromes with a more complex process at the highest dose levels leading to the
cerbrovascular mediated events.

354. Compared to a stable version of an element, an isotope of an element has:

A. the same atomic weight, but a different number of protons.


B. the same number of neutrons.
C. a different number of electrons.
D. a different atomic weight.

Correct answer is D. RATIONALE: This is the definition of an isotope, with a different number
of neutrons, and hence, a different atomic weight.

355. Which of the following statements about the effectiveness of 131I for treating thyroid
cancer is true?

A. Medullary carcinoma is treated more effectively than Hürthle cell carcinoma.


B. The follicular variant of papillary carcinoma is treated more effectively than Hürthle
cell carcinoma.
C. Tall cell carcinoma is treated more effectively than the follicular variant of papillary
carcinoma.
D. Insular carcinoma is treated more effectively than the follicular variant of papillary
carcinoma.

Correct answer is B. RATIONALE: The follicular variant of papillary carcinoma concentrates


radioiodine as well as any other well-differentiated variant. The other variants listed above are
unfavorable because they concentrate radioiodine poorly or not at all. Many experts do not
recommend the use of radioiodine therapy for Hürthle, tall, insular, or medullary carcinoma.

98 American College of Radiology


356. Which of the following statements about imaging findings associated with noninvasive
breast cancer is true?

A. MRI is better than mammography in detecting the extent of DCIS.


B. A segmental enhancement pattern on MRI is most commonly associated with a
diagnosis of DCIS.
C. A biopsy is not required if MRI does not confirm a suspicious finding of
microcalcification on mammography.
D. Mammographic patterns of DCIS do not indicate the probable aggressiveness of the
tumor.

THIS ITEM WAS NOT SCORED. RATIONALE: MRI has been shown to be more sensitive than
mammography in detection of additional sites of invasive and non-invasive breast cancer. The
most common mammographic appearances of calcification caused by DCIS are
linear/branching, coarse granular or fine granular calcifications. The linear findings represent
casts of calcifications in necrotic tumor along the ducts. T here is much overlap between
mammographic appearance and nuclear grade of tumor. Mammography cannot predict
pathology. However, mammography patterns can indicate the probable aggressiveness of the
tumor. Casting calcifications are more commonly associated with necrosis, and therefore, are a
more aggressive tumor. A mammographic mass indicates a higher likelihood of a
well-differentiated cancer or a lower grade lesion. MRI will demonstrate DCIS most commonly
as a linear/ductal pattern of enhancement. Segmental, regional, mass, or diffuse
enhancements are seen less often. MRI does not image calcification. Mammographically
evident suspicious microcalcifications without corresponding MRI enhancement should still be
biopsied through traditional methods. Reference: E.A. Morrison and L. Liberman, eds. Breast
MRI – Diagnosis and Intervention. 2005; Springer Press.

357. Which of the following equations is used to determine specificity?

A. True positive / (True positive + false negative)


B. True positive / (True positive + false positive)
C. True negative / (True negative + false negative)
D. True negative / (True negative + false positive)

Correct answer is D. RATIONALE: By definition, specificity is true negative / true negative +


false positive.

358. Which of the following processes is NOT commonly involved in the development of bone
metastases?

A. Avascular necrosis
B. Activation of osteoclasts
C. Cell adhesion molecules
D. Chemotaxis of metastatic cancer cells

Correct answer is A. RATIONALE: Avascular necrosis is not typically involved in bone


metastases.

Radiation Oncology In-Training Exam 2007 99


359. Microinvasion of DCIS is most likely to be found in patients with:

A. micropapillary histology.
B. DCIS and LCIS mixed histology.
C. a unilateral clear nipple discharge.
D. a clinically palpable mass >2.5 cm.

Correct answer is D. RATIONALE: Microinvasion is most likely to be found in patients with


DCIS lesions that are greater than 2.5 cm in diameter, those presenting with palpable masses
or (bloody) nipple discharge, and those with high-grade ductal carcinoma or comedo-necrosis.
Reference: Burstein, HJ, et al. Ductal carcinoma in situ of the breast. New England Journal of
Medicine (NEJM). 2004;350:1430-1441.

360. What is the most common site of metastasis for patients with soft tissue sarcoma?

A. Regional lymph nodes


B. Bone
C. Lung
D. Liver

Correct answer is C. RATIONALE: Most metastases occur in the lung. Regional lymph node
involvement occurs in <15% of cases.

100 American College of Radiology

You might also like